Sunteți pe pagina 1din 54

Part 1 : 11/11/10 07:46:17

Question 1 - CMA 1280 4-3 - Short-term Liquidity


Depoole Company is a manufacturer of industrial products and employs a calendar year for financial reporting
purposes. Assume that total quick assets exceeded total current liabilities both before and after each transaction
described. Further assume that Depoole has positive profits during the year and a credit balance throughout the year
in its retained earnings account.
The collection of a current accounts receivable of $29,000 would
A. Not affect the current or quick ratios.
B. Increase the current ratio.
C. Increase the quick ratio.
D. Decrease the current ratio and the quick ratio.
A. When a receivable is collected, there is no change in the quick or the current ratios. This is because this
transaction does not change the level of current assets or current liabilities. When a receivable is collected,
the company is simply exchanging one current and quick asset for another current and quick asset, of the
same amount. Therefore, the current and quick ratios do not change.
B. The collection of a receivable will not change the current ratio. See the correct answer for a complete explanation.
C. The collection of a receivable will not change the quick ratio. See the correct answer for a complete explanation.
D. The collection of a receivable will not change the current or quick ratios. See the correct answer for a complete
explanation.

Question 2 - CMA 688 4-3 - Short-term Liquidity


The data presented below shows actual figures for selected accounts of McKeon Company for the fiscal year ended
May 31, 2004, and selected budget figures for the 2005 fiscal year. McKeon's controller is in the process of reviewing
the 2005 budget and calculating some key ratios based on the budget. McKeon Company monitors yield or return
ratios using the average financial position of the company. (Round all calculations to three decimal places if
necessary.)
5/31/05 5/31/04
Current assets
$210,000 $180,000
Noncurrent assets
275,000 255,000
Current liabilities
78,000 85,000
Long-term debt
75,000 30,000
Common stock ($30 par value) 300,000 300,000
Retained earnings
32,000 20,000
2005 Operations
Sales*
$350,000
Cost of goods sold
160,000
Interest expense
3,000
Income taxes (40% rate)
48,000
Dividends declared and paid in 2005 60,000
Administrative expense
67,000
*All sales are credit sales.
Composition of Current Assets
5/31/05

5/31/04
(c) HOCK international, page 1

Part 1 : 11/11/10 07:46:17


Cash
$20,000 $10,000
Accounts receivable 100,000 70,000
Inventory
70,000 80,000
Other
20,000 20,000
$210,000 $180,000
The 2005 accounts receivable turnover for McKeon Company is
A. 1.882
B. 4.118
C. 5.000
D. 3.500
A. This answer is incorrect. See the correct answer for a full explanation.
B. Accounts receivable turnover is net credit sales divided by average accounts receivable. For McKeon,
budgeted credit sales for 2005 are $350,000 and the average accounts receivable is $85,000 ($70,000 actual at
the end of 2004 and $100,000 projected at the end of 2005). This gives us $350,000 $85,000 = 4.118.
C. This answer is incorrect. See the correct answer for a full explanation.
D. This answer uses the expected 2005 ending accounts receivable in the calculation instead of the average accounts
receivable balance for 2005.

Question 3 - CPA 0594 FARE Q49 - Earnings-based Analysis


On December 1, 2005, Clay Co. declared and issued a 6% stock dividend on its 100,000 shares of outstanding
common stock. There was no other common stock activity during 2005. What number of shares should Clay use in
determining earnings per share for 2005?
A. 106,000
B. 103,000
C. 100,000
D. 100,500
A. Despite the fact that the stock dividend took place only on December 1, we treat all shares that are issued
as part of a stock dividend or stock split to have been outstanding for the entire period. Therefore, these 6,000
shares issued as the stock dividend are considered to be outstanding for the entire year and this gives the
company 106,000 shares outstanding.
B. This answer is incorrect. See the correct answer for a complete explanation.
C. This answer fails to take into account the 6,000 shares that were issued as a stock dividend. See the correct
answer for a complete explanation.
D. This answer treats the shares that were issued as part of the stock dividend as having been outstanding for only 1
month. However, shares that are issued as a stock dividend or a stock split are treated as if they were outstanding the
entire year. See the correct answer for a complete explanation.

Question 4 - CMA 1291 P2 Q21 - Earnings-based Analysis

(c) HOCK international, page 2

Part 1 : 11/11/10 07:46:17


Sands, Inc. uses a calendar year for financial reporting. The company is authorized to issue 5,000,000 shares of $10
par common stock. At no time has Sands issued any potentially dilutive securities. Listed below is a summary of
Sands' common stock activities.
Number of common shares issued and outstanding at Dec. 31, 2003:1,000,000
Shares issued as a result of a 10% stock dividend on Sept. 30, 2004: 100,000
Shares issued for cash on March 31, 2005:
1,000,000
Number of common shares issued and outstanding at Dec. 31, 2005:2,100,000
A two-for-one stock split of Sands' common stock took place on March 31, 2006.
The weighted-average number of common shares to be used on computing earnings per common share for 2005 on
the 2006 comparative income statement is
A. 2,100,000.
B. 1,850,000.
C. 4,200,000.
D. 3,700,000.
A. This is the number of common shares outstanding as of December 31, 2005.
B. This is the number of shares used in the 2005 financial statements for the 2005 EPS calculation. This number
needs to be adjusted for the stock split that took place in 2006.
C. This is the number of common shares outstanding as of December 31, 2005 multiplied by 2. However, the
weighted-average number of common shares outstanding is different from the year-end number of shares outstanding.
D.
In the 2006 financial statements, the 2005 EPS calculations will have to include the effect of any stock splits
or stock dividends that occurred in 2006. In 2006 there was a 2-for-1 stock split. In order to properly compare
the EPSs of the different years, the company must go back and multiply the 2005 weighted-average shares
outstanding by 2.
At the end of 2004 there were 1,100,000 shares outstanding. This was the January 1, 2004 beginning amount
of 1,000,000 shares and the 100,000 shares that were issued in September 2004 as a stock dividend. Shares
issued as a stock dividend are considered to have been outstanding from January 1 of the first year
presented.
To this, the company needs to add the effect of any 2005 share transactions. The only transaction was the
issuance of 1,000,000 shares on March 31. Since these shares were outstanding for only 9 months of the year,
they will count as only 75% (9/12) for the weighted-average share calculation. These shares will become
750,000 shares for this calculation, giving the company 1,850,000 shares outstanding as of December 31,
2005, on the 2005 comparative income statement.
Since a two-for-one stock split took place on March 31, 2006, the 2005 weighted-average number of shares
outstanding needs to be multiplied by 2 when reporting the 2005 weighted-average number of shares
outstanding on the 2006 comparative income statements. Therefore, the weighted-average number of shares
outstanding for 2005 on the 2006 comparative statements is 3,700,000.

Question 5 - CMA 688 4-11 - Capital Structure and Solvency


(c) HOCK international, page 3

Part 1 : 11/11/10 07:46:17


A measure of long-term debt-paying ability is a company's
A. Times interest earned.
B. Return on assets.
C. Length of the operating cycle.
D. Inventory turnover.
A. Times interest earned is calculated as income before interest and taxes divided by the interest that the
company incurs. This does measure the ability of the company to pay its debt.
B. The return on assets does not measure the ability of a company to pay its debt.
C. The length of the operating cycle does not measure the ability of a company to pay its debt.
D. The inventory turnover does not measure the ability of a company to pay its debt.

Question 6 - CMA 685 4-14 - Profitability Analysis


Book value per common share represents the amount of shareholders' equity assigned to each outstanding share of
common stock. Which one of the following statements about book value per common share is correct?
A. A market price per common share that is greater than book value per common share is an indication of an
overvalued stock.
B. Book value per common share is the amount that would be paid to shareholders if the company were sold to
another company.
C. Book value per common share can be misleading because it is based on historical cost.
D. Market price per common share usually approximates book value per common share.
A. The market and book value of the shares may be different and a greater market price does not indicate an
overvalued share. This may happen if the company owns an asset that has appreciated greatly in value while the
company has held it. This asset will be recorded on the boos at its lower cost of acquisition and this will lead to an
understated book value per share.
B. The amount that would be paid to a shareholder in the event that the company is sold to another would be
determined by the specifics of the transaction itself, not by the book value per share.
C. Because the calculation of book value per share is based off of balance sheet amounts, it is very possible
that the book value per share will not reflect the current situation of the company. This is demonstrated by
assuming that a company owns an asset that has appreciated greatly in value while the company has held it.
This asset will be recorded on the boos at its lower cost of acquisition and this will lead to an understated
book value per share.
D. The market value and the book may approximate each other, but they do not have to. If the company has an asset
that has appreciated greatly in value, this increased value of the asset will not be reflected in book value per share,
but the market will have taken it into account when setting the market price of the shares.

Question 7 - CMA 1294 2-23 - Short-term Liquidity


The following inventory and sales data are available for the current year for Volpone Company. Volpone uses a
365-day year when computing ratios.

(c) HOCK international, page 4

Part 1 : 11/11/10 07:46:17

Net credit sales


Gross receivables
Inventory
Cost of goods sold

November 30, 2005 November 30,2004


$6,205,000
350,000
320,000
960,000
780,000
4,380,000

Volpone Company's average number of days to sell inventory for the current year is
A. 51.18 days.
B. 65.00 days.
C. 80.00 days.
D. 72.50 days.
A. The average number of days to sell inventory is calculated as 365 divided by the inventory turnover. Inventory
turnover is calculated as the cost of goods sold divided by average inventory. This answer uses sales instead of the
cost of goods sold to calculate the inventory turnover.
B. The average number of days to sell inventory is calculated as 365 divided by the inventory turnover. Inventory
turnover is calculated as the cost of goods sold divided by average inventory. This answer uses only beginning
inventory in the calculation of inventory turnover.
C. The average number of days to sell inventory is calculated as 365 divided by the inventory turnover. Inventory
turnover is calculated as the cost of goods sold divided by average inventory. This answer uses only ending inventory
in the calculation of inventory turnover.
D. The average number of days to sell inventory is calculated as 365 divided by the inventory turnover.
Inventory turnover is calculated as the cost of goods sold divided by average inventory. Cost of goods sold
was $4,380,000 and average inventory was $870,000. This gives an inventory turnover of 5.03. Dividing 365 by
5.03 gives us 72.5 days as the number of days to sell inventory.

Question 8 - CMA 1295 2-21 - Other Analytical Issues


In assessing the financial prospects for a firm, financial analysts use various techniques. An example of vertical,
common-size analysis is
A. Advertising expense increased by 3% over the previous year.
B. Advertising expense is 4% of sales.
C. An assessment of the relative stability of a firm's level of vertical integration.
D. A comparison in financial ratio form between two or more firms in the same industry.
A. Vertical, common-size analysis creates financial statements in which each component is measured as a
percentage of another element of the financial statements for that same period. A comparison between periods for the
same company is not vertical, common-size analysis. This is horizontal, common-size analysis.
B. Vertical, common-size analysis creates financial statements in which each component is measured as a
percentage of another element of the financial statements for that same period. For example, all items on the
balance sheet are measured as a percentage of total assets and all income statement items are measured as
a percentage of total sales. Advertising expense being 2% of sales is such an example of vertical,
common-size analysis.
C. Vertical, common-size analysis creates financial statements in which each component is measured as a
percentage of another element of the financial statements for that same period. Vertical integration occurs when a
company owns other companies that are suppliers or customers. Vertical integration is not related to vertical,
common-size analysis.

(c) HOCK international, page 5

Part 1 : 11/11/10 07:46:17


D. Vertical, common-size analysis creates financial statements in which each component is measured as a
percentage of another element of the financial statements for that same period. A comparison between companies
within the same industry is not vertical, common-size analysis.

Question 9 - CIA 1194 IV-16 - Other Analytical Issues


A company has a 50% gross margin, general and administrative expenses of $50, interest expense of $20, and net
income of $10 for the year just ended. If the corporate tax rate is 50%, the level of sales revenue for the year just
ended was
A. $135
B. $150
C. $90
D. $180
A. This answer is incorrect. See the correct answer for a complete explanation.
B. This answer is incorrect. See the correct answer for a complete explanation.
C. This is the amount of the cost of goods sold. See the correct answer for a complete explanation.
D. If net income is $10 and the tax rate is 50%, we know that income before taxes is $20. We also know that
the cost of goods sold is 50% of the sales price since the gross margin is 50%. Total expenses are $70
(general and administrative and interest). The formula for the calculation of the revenue is as follows (where R
represents revenue): R - .5R - 70 = 20. Solving for R we get $180, which is the revenue of this company.

Question 10 - CMA 1287 4-1 - Other Analytical Issues


When a balance sheet amount is related to an income statement amount in computing a ratio,
A. The income statement amount should be converted to an average for the year.
B. The balance sheet amount should be converted to an average for the year.
C. Both amounts should be converted to market value.
D. Comparisons with industry ratios are not meaningful.
A. Because the income statement records the effects of transactions over a period of time, they cannot be averaged.
B. When balance sheet amounts are used together with an income statement amount, the balance sheet
amount used should be an average for the year. This is important because it prevents the ratio from being
distorted by an unusually high or low balance sheet amount at the end of the year.
C. This answer is incorrect. There is no market value of an income statement item and it may be very difficult, if not
impossible, to convert some balance sheet items to a market value.
D. Comparisons with industry ratios are meaningful, even in the situation in which the ratio compares a balance sheet
amount to an income statement amount.

Question 11 - CMA 1293 2-18 - Earnings-based Analysis

(c) HOCK international, page 6

Part 1 : 11/11/10 07:46:17


An increase in the market price of a company's common stock will immediately affect its
A. Earnings per share.
B. Debt to equity ratio.
C. Dividend yield.
D. Dividend payout ratio.
A. Earnings per share is not affected by the market price of the company's shares.
B. The total debt-to-equity ratio is not affected by the market price of the company's shares.
C. The dividend yield ratio is calculated as the cash dividend divided by the market price of the share. As the
market price of the shares increases, the dividend yield will decrease.
D. The dividend payout ratio is not affected by the market price of the company's shares.

Question 12 - CMA 688 4-4 - Short-term Liquidity


The data presented below shows actual figures for selected accounts of McKeon Company for the fiscal year ended
May 31, 2004, and selected budget figures for the 2005 fiscal year. McKeon's controller is in the process of reviewing
the 2005 budget and calculating some key ratios based on the budget. McKeon Company monitors yield or return
ratios using the average financial position of the company. (Round all calculations to three decimal places if
necessary.)
5/31/05 5/31/04
Current assets
$210,000 $180,000
Noncurrent assets
275,000 255,000
Current liabilities
78,000 85,000
Long-term debt
75,000 30,000
Common stock ($30 par value) 300,000 300,000
Retained earnings
32,000 20,000
2005 Operations
Sales*
$350,000
Cost of goods sold
160,000
Interest expense
3,000
Income taxes (40% rate)
48,000
Dividends declared and paid in 2005 60,000
Administrative expense
67,000
*All sales are credit sales.
Composition of Current Assets
5/31/05 5/31/04
Cash
$20,000 $10,000
Accounts receivable 100,000 70,000
Inventory
70,000 80,000
20,000 20,000
Other
$210,000 $180,000
Using a 365-day year, McKeon's days of sales in inventory is
A. 78 days.
B. 183 days.
(c) HOCK international, page 7

Part 1 : 11/11/10 07:46:17


C. 160 days.
D. 171 days.
A. This answer is incorrect. See the correct answer for a full explanation.
B. In order to calculate the days sales in inventory for 2005, we first need to calculate how many times the inventory
turns over during the year. This is COGS divided by average inventory. This answer uses the 2004 year-end inventory
instead of the average inventory for 2005 in the calculation of the number of days of sales in inventory.
C. In order to calculate the days sales in inventory for 2005, we first need to calculate how many times the inventory
turns over during the year. This is COGS divided by average inventory. This answer uses the year-end inventory
instead of the average inventory in the calculation of the number of days of sales in inventory.
D. In order to calculate the days sales in inventory for 2005, we first need to calculate how many times the
inventory turns over during the year. This is COGS divided by average inventory. COGS is budgeted at
$160,000 and average inventory is $75,000 ($70,000 actual at year-end 2004 and $80,000 planned for year-end
2005). This gives us an inventory turnover ratio of $160,000 $75,000 = 2.13 times per year. If the inventory
turns over 2.13 times during the year, then the days of sales in inventory equals 171 days (365 2.13).

Question 13 - CMA 1293 2-16 - Other Analytical Issues


The ratio that measures a firm's ability to generate earnings from its resources is
A. Asset turnover.
B. Sales to working capital.
C. Days' sales in inventory.
D. Days' sales in receivables.
A. Asset turnover is calculated as sales divided by total assets and is a measure of the company's ability to
generate earnings from all of its resources.
B. Though this ratio does measure the effectiveness of the use of working capital, it does not measure the
effectiveness of the use of all of the assets. This is because working capital includes only current assets.
C. This ratio measures how quickly the company is able to sell its inventory.
D. This ratio measures how effectively the company is collecting its receivables.

Question 14 - CMA 695 2-2 - Short-term Liquidity


CPZ Enterprises had the following account information.
Accounts receivable
Accounts payable
Bonds payable, due in 10 years
Cash
Interest payable, due in three months
Inventory
Land
Notes payable, due in six months

$200,000
80,000
300,000
100,000
10,000
400,000
250,000
50,000
(c) HOCK international, page 8

Part 1 : 11/11/10 07:46:17


Prepaid expences

40,000

The company has an operating cycle of five months.


What is the company's acid test (quick) ratio?
A. 1.68
B. 2.31
C. 2.14
D. 0.68
A. The quick ratio is calculated as quick assets (current assets excluding inventory and prepaid expenses) divided by
current liabilities. This answer includes inventory and prepaid expenses as quick assets and includes bonds payable
as a current liability.
B. The quick ratio is calculated as quick assets (current assets excluding inventory and prepaid expenses) divided by
current liabilities. This answer does not include the interest payable as a current liability.
C. The quick ratio is calculated as quick assets (current assets excluding inventory and prepaid expenses)
divided by current liabilities. Quick assets include accounts receivable ($200,000) and cash ($100,000).
Current liabilities include accounts payable ($80,000), interest payable ($10,000) and notes payable ($50,000).
Note that even though the notes payable are due after the operating cycle is over, the distinction for a current
asset or liability is that it will be converted or settled within 12 months or the operating cycle, whichever is
longer. The total current assets are $300,000 and current liabilities are $140,000. This gives us a quick ratio of
2.14.
D. The quick ratio is calculated as quick assets (current assets excluding inventory and prepaid expenses) divided by
current liabilities. This answer includes bonds payable as a current liability.

Question 15 - CMA 692 2-25 - Short-term Liquidity


The ratio of sales to working capital is a measure of
A. Profitability.
B. Liquidity.
C. Financial leverage.
D. Collectibility.
A. Sales and working capital do not measure the profitability of the company.
B. Liquidity relates to the ability of the company to meet its liabilities as they come due. Most ratios that
involve working capital are liquidity measures, as is this one.
C. Financial leverage looks at the use of debt and equity to capitalize the company and sales and working capital are
not used in the calculation of financial leverage.
D. Sales and working capital do not measure the ability of the company to collect its receivables.

Question 16 - CMA 685 4-21 - Earnings-based Analysis


Watson Corporation computed the following items from its financial records for the year just ended:

(c) HOCK international, page 9

Part 1 : 11/11/10 07:46:17


Price-earnings ratio 12
Payout ratio
.6
Asset turnover
.9
The dividend yield on Watson's common stock is
A. 7.2%
B. 7.5%
C. 5.0%
D. 10.8%
A. This answer simply multiplies the P-E ratio by the payout ratio. See the correct answer for a complete explanation.
B. This answer simply multiplies the P-E ratio by the asset turnover. See the correct answer for a complete
explanation.
C. The dividend yield is calculated as the cash dividend per share divided by the market price per share. Since
we are not given these items directly, we will need to calculate them from the information given. Let us
assume that the P-E ratio is made up of a $12 market price and $1 of earnings. This gives us the P-E ratio of
12 that we are told the company has. The payout ratio tells us what percentage of earnings were distributed
as cash, and since it is .6, we know that $.60 was distributed to the shareholders. We now know the
necessary information to calculate the dividend yield. The market price is $12 and the cash dividend was .6.
This gives a dividend yield of 5%.
D. This answer is incorrect. See the correct answer for a complete explanation.

Question 17 - CIA 1190 IV-55 - Capital Structure and Solvency


Assume that a company's debt ratio is currently 50%. It plans to purchase fixed assets either by using borrowed funds
for the purchase or by entering into an operating lease. The company's debt ratio as measured by the balance sheet
will
A. Increase whether the assets are purchased or leased.
B. Remain unchanged whether the assets are purchased or leased.
C. Increase if the assets are purchased, and remain unchanged if the assets are leased.
D. Increase if the assets are purchased, and decrease if the assets are leased.
A. The debt ratio is calculated as the total debt divided by the total assets. If the assets is purchased by using
borrowed funds, the ratio will change. See the correct answer for a more detailed explanation.
B. The debt ratio is calculated as the total debt divided by the total assets. If the asset is purchased through the use of
borrowed funds, the ratio will change. See the correct answer for a more detailed explanation.
C. The debt ratio is calculated as the total debt divided by the total assets. Let us assume that currently the
company has $100 of debt and $200 of assets. This gives us the .50 ratio that currently exists. If the assets are
purchased, the debt and the assets will increase by the amount of the purchase (let us assume $50). This will
make the ratio $150 / $250, or .60 - an increase. If the assets are acquired under an operating lease there will
be no debt recorded and there will be no asset recorded, so the ratio will remain unchanged.
D. The debt ratio is calculated as the total debt divided by the total assets. If the asset is leased under an operating
lease, the ratio will remain unchanged as there will be no increases in the level of debt or the level of assets.

(c) HOCK international, page 10

Part 1 : 11/11/10 07:46:17


Question 18 - CPA 0592 FARE Q59 - Earnings-based Analysis
Timp, Inc. had the following common stock balances and transactions during 2005:
January 1 - Common stock outstanding: 30,000 shares
February 1 - Issued a 10% common stock dividend: 3,000 shares
March 1- Issued common stock for cash: 9,000 shares
July 1 - Issued common stock for cash: 8,000 shares
December 31- Common stock outstanding: 50,000 shares
What was Timp's 2005 weighted average shares outstanding?
A. 40,000
B. 44,250
C. 46,000
D. 44,500
A. This answer is incorrect. See the correct answer for a complete explanation.
B. This answer is incorrect. See the correct answer for a complete explanation.
C. This answer is incorrect. See the correct answer for a complete explanation.
D. In order to calculate the weighted average shares outstanding, we start with the 30,000 shares outstanding
on January 1. The 3,000 shares issued as a stock dividend are assumed to have been outstanding the entire
year, and are therefore 3,000 weighted average shares. The shares issued on March 1 were outstanding for
only 10 months and therefore need to be multiplied by 10/12 to calculate the weighted average number of
shares. This is 7,500. The 8,000 shares issued on July 1 were outstanding for only 6 months and therefore are
multiplied by 6/12, giving us 4,000 weighted average shares. Adding these figures together, we get 44,500
weighted average shares outstanding for 2005.

Question 19 - CIA 1193 IV-48 - Capital Structure and Solvency


A company is considering the early retirement of its 10%, 10-year bonds payable. Before retiring the bonds, the
company's capital structure was
Current liabilities
Long-term liabilities: Notes payable (due in 5 years)
Bonds payable
Premium on bonds payable
Owner's equity: Common stock ($5 par value)
Paid-in capital in excess of par
Retained earnings

$125,000
200,000
300,000
25,000
150,000
50,000
450,000

If the bonds can be retired at 103.5%, the


A. Asset turnover ratio will decrease.
B. Debt-equity ratio will increase.
C. Return on owner's equity will decrease.
D. Financial leverage will decrease.
A. Because of the retirement of the bonds is done by spending cash, the total assets of the company will decrease.
This decrease in total assets will increase the asset turnover ratio.

(c) HOCK international, page 11

Part 1 : 11/11/10 07:46:17


B. Because the amount of debt will be decreased and their will be no change in equity, the debt-equity ratio will
decrease.
C. Because the bonds will be retired there will be less interest expense which will increase net income. This increase
in net income will cause the return on owners equity to increase.
D. Financial leverage relates to the amount of debt that a company uses for its financing. As the amount of
debt decreases, the leverage of the company will decrease. The fact that the bonds will be retired at 103.5% of
the face value is not relevant to this question.

Question 20 - CMA 1280 4-5 - Profitability Analysis


Depoole Company is a manufacturer of industrial products and employs a calendar year for financial reporting
purposes. Assume that total quick assets exceeded total current liabilities both before and after each transaction
described. Further assume that Depoole has positive profits during the year and a credit balance throughout the year
in its retained earnings account.
The issuance of new shares in a five-for-one split of common stock
A. Decreases total shareholders' equity.
B. Increases the book value per share of common stock.
C. Increases total shareholders' equity.
D. Decreases the book value per share of common stock.
A. The issuance of shares through a stock split does not change the level of shareholders' equity accounts. There is
no journal entry to record a stock split, and thus the split does not affect shareholders' equity.
B. When a stock split is carried out, the book value per share will decrease due to the fact that there are more shares
outstanding and there is no increase in the equity accounts of the company.
C. The issuance of shares through a stock split does not change the level of shareholders' equity accounts. There is
no journal entry to record a stock split, and thus the split does not affect shareholders' equity.
D. Book value per share is calculated as the amount of common equity divided by the number of shares
outstanding. Therefore, a stock split will reduce the book value per share. This is because the stock split
increases the number of shares outstanding, but does not increase the common equity accounts at all.

Question 21 - CMA 695 2-1 - Short-term Liquidity


CPZ Enterprises had the following account information.
Accounts receivable
Accounts payable
Bonds payable, due in ten years

$200,000
80,000
10,000

Cash
100,000
Interest payable, due in three months 10,000
Inventory
400,000
Land
250,000
Notes payable, due in six months
50,000

(c) HOCK international, page 12

Part 1 : 11/11/10 07:46:17


Prepaid expenses

40,000

The company has an operating cycle of five months.


The current ratio for CPZ Enterprises is
A. 2.14
B. 5.00
C. 5.29
D. 1.68
A. The current ratio is calculated as current assets divided by current liabilities. This answer does not include the
inventory in the current assets.
B. The current ratio is calculated as current assets divided by current liabilities. This answer does not include the
prepaid expenses in the current assets.
C. The current ratio is calculated as current assets divided by current liabilities. Current assets include
accounts receivable ($200,000), cash ($100,000), inventory ($400,000) and prepaid expenses ($40,000). Current
liabilities include accounts payable ($80,000), interest payable ($10,000) and notes payable ($50,000). Note
that even though the notes payable are due after the operating cycle is over, the distinction for a current asset
or liability is that it will be converted or settled within 12 months or the operating cycle, whichever is longer.
The total current assets are $740,000 and current liabilities are $140,000. This gives us a current ratio of 5.29.
D. The current ratio is calculated as current assets divided by current liabilities. This answer includes the bonds
payable as a current liability.

Question 22 - CMA 1293 2-15 - Short-term Liquidity


A ratio that measures the movement of current assets is
A. Return on owners' equity.
B. Working capital turnover.
C. The current ratio.
D. Working capital to total assets.
A. This ratio is calculated as income divided by owners' equity. It does not provide a measure for the use of current
assets.
B. Working capital turnover is calculated as sales divided by average working capital. This measures the use
of the company's current assets to generate sales. A high turnover ratio is desired.
C. The current ratio simply measures the ratio between current assets and current liabilities and not how those current
assets are being used.
D. This measure would simply indicate what percentage of the company's assets are current assets, and not how
those assets are being used.

Question 23 - CMA 684 4-9 - Profitability Analysis


Return on investment (ROI) is a term often used to express income earned on capital invested in a business unit. A
company's ROI is increased if
(c) HOCK international, page 13

Part 1 : 11/11/10 07:46:17

A. Sales increase by the same dollar amount as expenses and total assets.
B. Net profit margin on sales increases by the same percentage as total assets.
C. Sales remain the same and expenses are reduced by the same dollar amount that total assets increase.
D. Sales decrease by the same dollar amount that expenses increase.
A.
ROI is calculated as net income divided by total assets. One way to solve this problem is to set up some actual
numbers for a basic ROI. For example, sales = $500,000, net income = $100,000 and total assets = $400,000. ROI =
$100,000 $400,000, or .25.
If we increase sales, expenses and total assets by the same amounts, for example by $50,000, the new amounts will
be: sales = $550,000, net income = $100,000 (unchanged because both sales and expenses increased by the same
amount), and total assets = $450,000. ROI will become $100,000 $450,000, which is .22 and is a decrease, not an
increase.
B.
ROI is calculated as net income divided by total assets. One way to solve this problem is to set up some actual
numbers for a basic ROI. For example, sales = $500,000, net income = $100,000 and total assets = $400,000. ROI =
$100,000 $400,000, or .25.
If we increase both the profit margin on sales and the assets of the company by the same percentage, say 10%, ROI
will remain unchanged. Net income will increase by 10% to $110,000 and total assets will increase by 10% to
$440,000. ROI will be $110,000 $440,000, which is unchanged at .25.
C.
ROI is calculated as net income divided by total assets. One way to solve this problem is to simply set up
some actual numbers for a basic ROI. For example, sales = $500,000, net income = $100,000 and total assets =
$400,000. ROI = $100,000 $400,000, or .25.
If we reduce expenses by $50,000 and increase total assets by $50,000, net income will increase by $50,000 to
$150,000 because sales remained the same while expenses were reduced. Total assets will increase by
$50,000 to $450,000. ROI will change to $150,000 $450,000, which is .33, an increase.

D.
ROI is calculated as net income divided by total assets. One way to solve this problem is to set up some actual
numbers for a basic ROI. For example, sales = $500,000, net income = $100,000 and total assets = $400,000. ROI =
$100,000 $400,000, or .25.
If we decrease sales and increase expenses by $25,000 each, sales will decrease to $475,000, expenses will
increase to $425,000, and net income will fall to $50,000. ROI will become $50,000 $400,000, which is lower.

Question 24 - CMA 690 4-21 - Other Analytical Issues


Assume the information below for Ramer Company, for Matson Company, and for their common industry represents a
recent year.
Industry
Ramer Matson Average
(c) HOCK international, page 14

Part 1 : 11/11/10 07:46:17


Current ratio
Accounts receivable turnover
Inventory turnover
Times interest earned
Debt-to-equity ratio
Return on investment
Dividend payout ratio
Earnings per share

3.50
2.80
5.00
8.10
6.20
8.00
9.00 12.30
0.70
0.40
0.15
0.12
0.80
0.60
$3.00 $2.00

3.00
6.00
6.10
10.40
0.55
0.15
0.55
--

Some of the ratios and data for Ramer and Matson are affected by income taxes. Assuming no interperiod income tax
allocation, which of the following items would be directly affected by income taxes for the period?
A. Accounts receivable turnover and inventory turnover.
B. Return on investment and earnings per share.
C. Debt/equity ratio and dividend payout ratio.
D. Current ratio and debt/equity ratio.
A. Because neither of these ratios are based on net income after taxes, the level of taxes would not impact these
ratios.
B. Both of these would be affected by taxes because they both use after tax profits in their calculation.
C. The debt-to-equity ratio does not use net income after taxes so it is not affected by the income taxes for the period.
D. Because neither of these ratios are based on net income after taxes, the level of taxes would not impact these
ratios.

Question 25 - CMA 1280 4-1 - Short-term Liquidity


Depoole Company is a manufacturer of industrial products and employs a calendar year for financial reporting
purposes. Assume that total quick assets exceeded total current liabilities both before and after each transaction
described. Further assume that Depoole has positive profits during the year and a credit balance throughout the year
in its retained earnings account.
Payment of a trade account payable of $64,500 would
A. Increase the quick ratio but the current ratio would not be affected.
B. Increase both the current and quick ratios.
C. Decrease both the current and quick ratios.
D. Increase the current ratio but the quick ratio would not be affected.
A. The current ratio would be affected by this transaction as both the numerator and denominator of the current ratio
would be decreased by the amount of the liability settled.
B. Because Depoole has a quick and current ratio of greater than one, the payment of a payable would
increase both ratios. This is because the payment of a payable will reduce both the current assets and the
current liabilities by the same amount. Let us assume that Depoole had current and quick assets of $150 and
current liabilities of $100. Given that the current and quick assets were both $150, both ratios are 1.50. Now
let assume that they paid a $20 liability. This reduced the quick and current assets to $130, and the current
liabilities to $80. Now, both ratios are 1.625, which is higher than they had been.
C. Because both of these ratios were higher than 1 prior to this transaction, both of these ratios will increase as a
result of the settlement of the liability.

(c) HOCK international, page 15

Part 1 : 11/11/10 07:46:17


D. The quick ratio would be affected by this transaction as both the numerator and denominator of the quick ratio
would be decreased by the amount of the liability settled.

Question 26 - CMA 693 2-3 - Short-term Liquidity


Lisa, Inc.
Statement of Financial Position
December 31, 2005
(in thousands)
2005 2004
Assets - Current
Cash
Trading securities
Accounts receivable (net)
Inventories (at lower of cost of market)
Prepaid items
Total Current Assets
Assets - Long-Term
Long-term investments Securities (at lost)
Property, plant & equipment Land (at cost)
Building (net)
Equipment (net)
Intangible assets Patents (net)
Goodwill (net)
Total Long-Term Assets
Total Assets
Liabilities - Current
Notes payable
Accounts payable
Accrued interest
Total current liabilities
Liabilities - Long-Term
Notes payable 10% due 12/31/2005
Bonds payable 12% due 12/31/2004
Total long-term debt
Total liabilities
Equity
Preferred - 5% cumulative, $100 par, nonparticipating, authorized, issued and
outstanding, 1,000 shares
Common - $10 par 20,000 shares authorized, 15,000 issued and outstanding shares
Additional paid-in capital - common
Retained earnings
Total Equity
Total Liabilities & Equity

$30
20
45
60
15
170

$25
15
30
50
20
140

25
75
80
95
35
20
330
$500

20
75
90
100
17
13
315
$455

$23
47
15
85

$12
28
15
55

10
15
25
110

10
15
25
80

100 100
150
75
65
390
$500

150
75
50
375
$455

Assume net credit sales were $300,000 for 2005, Lisa Inc.'s average collection period for 2005, using a 360-day year
was
A. 54 days.
B. 36 days.
C. 61 days.
D. 45 days.
(c) HOCK international, page 16

Part 1 : 11/11/10 07:46:17


A. This answer is incorrect. See the correct answer for a complete explanation.
B. This answer is incorrect. See the correct answer for a complete explanation.
C. This answer is incorrect. See the correct answer for a complete explanation.
D. The average collection period is calculated as average accounts receivable divided by the average daily
sales. Average receivables were 37,500 (45,000 + 30,000 divided by 2) and the average sales were $833.33
($300,000 divided by 360). Dividing $37,500 by $833.33 gives us 45 days of sales in receivables. This means
that it takes 45 days to collect receivables.

Question 27 - CMA 688 4-12 - Short-term Liquidity


Using the data presented below, calculate the cost of sales for the Beta Corporation for the past year.
Current ratio
3.5
Acid test ratio
3.0
Current liabilities at year end $600,000
Beginning inventory
$500,000
Inventory turnover
8.0
A. $6,400,000
B. $1,600,000
C. $2,400,000
D. $3,200,000
A. This answer is incorrect. See the correct answer for a detailed explanation of how this problem should be solved.
B. This answer is incorrect. See the correct answer for a detailed explanation of how this problem should be solved.
C. This answer simply multiplies the ending inventory (how this is calculated is shown in the correct answer
explanation) by the inventory turnover number.
D. We first need to start this question by calculating what the inventory balance is. If the Current liabilities are
$600,000 and the current ratio is 3.5, that means that current assets are $2,100,000. Given an acid test ratio of
3.0 and current liabilities of $600,000, we know that the numerator of the quick ratio is $1,800,000. The
difference between the current ratio and the quick ratio is inventory, so the difference in the numerator of
these two items must be inventory. Thus, ending inventory is $300,000. Since ending inventory was $300,000
and beginning inventory was $500,000, we have an average inventory of $400,000. Inventory turnover is
calculated as cost of sales divided by average inventory. Using the inventory turnover of 8, we know that the
sales were 8 times the average inventory, or $3,200,000.

Question 28 - CMA 693 2-1 - Short-term Liquidity


Lisa, Inc.
Statement of Financial Position
December 31, 2005
(in thousands)
2005 2004
Assets - Current
(c) HOCK international, page 17

Part 1 : 11/11/10 07:46:17


Cash
Trading securities
Accounts receivable (net)
Inventories (at lower of cost of market)
Prepaid items
Total Current Assets
Assets - Long-Term
Long-term investments Securities (at lost)
Property, plant & equipment Land (at cost)
Building (net)
Equipment (net)
Intangible assets Patents (net)
Goodwill (net)
Total Long-Term Assets
Total Assets
Liabilities - Current
Notes payable
Accounts payable
Accrued interest
Total current liabilities
Liabilities - Long-Term
Notes payable 10% due 12/31/2005
Bonds payable 12% due 12/31/2004
Total long-term debt
Total liabilities
Equity
Preferred - 5% cumulative, $100 par, nonparticipating, authorized, issued and
outstanding, 1,000 shares
Common - $10 par 20,000 shares authorized, 15,000 issued and outstanding shares
Additional paid-in capital - common
Retained earnings
Total Equity
Total Liabilities & Equity

$30
20
45
60
15
170

$25
15
30
50
20
140

25
75
80
95
35
20
330
$500

20
75
90
100
17
13
315
$455

$23
47
15
85

$12
28
15
55

10
15
25
110

10
15
25
80

100 100
150
75
65
390
$500

150
75
50
375
$455

Lisa Inc.'s acid test (quick) ratio at December 31, 2005 was
A. .6 : 1.0
B. 2.0 : 1.0
C. 1.1 : 1.0
D. 1.8 : 1.0
A. This answer does not include the receivables in the numerator of the calculation.
B. This answer includes all current assets in the calculation.
C. The acid test (or quick) ratio is calculated as follows: (Cash + Receivables + Trading Securities) / Current
Liabilities. Given the information in this question, we get ($30,000 + $20,000 + $45,000) / $85,000. This is 1.1.
D. This answer incorrectly includes inventory in the calculation of the numerator.

Question 29 - CMA 1294 2-24 - Short-term Liquidity


The following inventory and sales data are available for the current year for Volpone Company. Volpone uses a
(c) HOCK international, page 18

Part 1 : 11/11/10 07:46:17


365-day year when computing ratios.

Net credit sales


Gross receivables
Inventory
Cost of goods sold

November 30, 2005 November 30,2004


$6,205,000
350,000
320,000
960,000
780,000
4,380,000

Volpone Company's operating cycle for the current year is


A. 86.17 days.
B. 70.61 days.
C. 92.21 days.
D. 98.87 days.
A. This answer is incorrect. See the correct answer for a complete explanation.
B. This answer is incorrect. See the correct answer for a complete explanation.
C. The operating cycle is calculated as the average time that inventory is held before sale plus the average
time the receivables are held before they are collected. The average number of days to sell inventory is
calculated as 365 divided by the inventory turnover. Inventory turnover is calculated as the cost of goods sold
divided by average inventory. Cost of goods sold was $4,380,000 and average inventory was $870,000. This
gives an inventory turnover of 5.03. Dividing 365 by 5.03 gives us 72.5 days as the number of days to sell
inventory. The average number of days to collect receivables is calculated as 365 divided by the receivables
turnover. Receivables turnover is calculated as credit sales divided by average receivables. Credit sales were
$6,205,000 and average receivables were $335,000. This gives a receivables turnover of 18.52. Dividing 365 by
18.52 gives us 19.71 days as the number of days to collect receivables. Adding these two periods together,
we get an operating cycle of 92.2 days.
D. This answer is incorrect. See the correct answer for a complete explanation.

Question 30 - CPA 0592 FARE1 Q60a - Earnings-based Analysis


On June 30, 2004, Lomond, Inc. issued twenty, $10,000, 7% bonds at par. Each bond was convertible into 200 shares
of common stock. On January 1, 2005, 10,000 shares of common stock were outstanding. None of the bonds were
converted during the period. The following amounts were reported in Lomond's income statement for the year ended
December 31, 2005:
Revenues
Operating expenses
Interest on bonds
Income before income tax
Income tax at 30%
Net income

$977,000
$(920,000)
$(14,000)
$43,000
$(12,900)
$30,100

What amount should Lomond report as its 2005 basic earnings per share?
A. $4.30
B. $2.85
C. $2.50
D. $3.01
A. This answer uses the before tax income to calculate BEPS. The net income should be after income taxes. See the
(c) HOCK international, page 19

Part 1 : 11/11/10 07:46:17


correct answer for a complete explanation.
B. This answer is incorrect. See the correct answer for a complete explanation.
C. This answer is incorrect. See the correct answer for a complete explanation.
D. Basic earnings per share (BEPS) is calculated as the income available to common shareholders divided by
the weighted average number of common shares outstanding during the period. For BEPS we do not worry
about the convertible bonds as those shares were not outstanding during the period. The convertible bonds
will be addressed in Diluted EPS. The income available to common shareholders was $30,100 and there were
10,000 shares outstanding. This gives us a BEPS of $3.01.

Question 31 - CMA 685 4-19 - Other Analytical Issues


Cascade Company had sales of $300,000 in 2003 and the price index for its industry is expected to rise from 300 in
2003 to 320 in 2004. The level of sales that Cascade must reach in 2004 in order to achieve a real growth rate of 20%
is
A. $360,000
B. $320,000
C. $384,000
D. $337,500
A. This answer does not take into account the increase in the price index. However, since the real growth rate is
calculated using a constant dollar, the increase in the price index must be included in order to determine the sales that
will be required to achieve a real growth rate of 20%.
B. This answer is simply the increase in sales that will occur if the company does nothing. This will occur simply
because of an increase in the price level. In order to achieve a 20% real growth rate, the sales will need to increase by
20% in constant dollar terms.
C. The real growth rate is growth that is stated in a constant dollar terms. Because the price index indicates
that their sales will increase from $300,000 to $320,000 in value simply because of an increase in prices, and
not because of an increase in quantity sold, their sales will need to increase by 20% of this amount in order to
have a real increase. 20% of $320,000 is $64,000. Therefore, in order to have a real growth of 20%, sales will
need to increase to $384 in 2004.
D. This answer is incorrect. See the correct answer for a complete explanation.

Question 32 - CMA 688 4-6 - Return on Invested Capital


The data presented below shows actual figures for selected accounts of McKeon Company for the fiscal year ended
May 31, 2004, and selected budget figures for the 2005 fiscal year. McKeon's controller is in the process of reviewing
the 2005 budget and calculating some key ratios based on the budget. McKeon Company monitors yield or return
ratios using the average financial position of the company. (Round all calculations to three decimal places if
necessary.)

Current assets
Noncurrent assets
Current liabilities

5/31/05 5/31/04
$210,000 $180,000
275,000 255,000
78,000 85,000

(c) HOCK international, page 20

Part 1 : 11/11/10 07:46:17


Long-term debt
75,000 30,000
Common stock ($30 par value) 300,000 300,000
Retained earnings
32,000 20,000
2005 Operations
Sales*
$350,000
Cost of goods sold
160,000
Interest expense
3,000
Income taxes (40% rate)
48,000
Dividends declared and paid in 2005 60,000
Administrative expense
67,000
*All sales are credit sales.
Composition of Current Assets
5/31/05 5/31/04
Cash
$20,000 $10,000
Accounts receivable 100,000 70,000
Inventory
70,000 80,000
20,000 20,000
Other
$210,000 $180,000
The 2005 return on assets for McKeon Company is
A. 0.166
B. 0.261
C. 0.148
D. 0.157
A. This answer is incorrect. See the correct answer for a complete explanation.
B. This answer is incorrect. See the correct answer for a complete explanation.
C. This answer is incorrect. See the correct answer for a complete explanation.
D. The return on assets is net income after taxes and interest divided by average total assets of $460,000
($485,000 in 2005 and $435,000 in 2004). Net income is $72,000 ($350,000 - $160,000 - $3,000 - $48,000 $67,000). This gives us $72,000 $460,000 = .156. Dividends are not included in the calculation of net income
because they are a distribution of that income.

Question 33 - CMA 0694 P2 Q15 - Earnings-based Analysis


At the beginning of the fiscal year, June 1, 2005, Boyd Corporation had 80,000 shares of common stock outstanding.
Also outstanding was $200,000 of 8% convertible bonds that had been issued at $1,000 par. The bonds were
convertible into 20,000 shares of common stock; however, no bonds were converted during the year. The company's
tax rate is 34%, and the Aa bond interest rate has been 10%. Boyd's net income for the year was $107,000. The fully
diluted earnings per share (DEPS - rounded to the nearest cent) of Boyd common stock for the fiscal year ended May
31, 2005 was
A. $1.20
B. $1.18
C. $1.07
D. $1.12

(c) HOCK international, page 21

Part 1 : 11/11/10 07:46:17


A. This answer incorrectly uses the market rate to calculate the interest on the bonds. For purposes of calculating
DEPS, we need to know only the stated rate for the bond because this determines how much cash was paid, which
will be the amount of the tax deduction for interest for the company. See the correct answer for a complete
explanation.
B. In order to calculate DEPS, we need to assume that the bonds had been converted at the beginning of the
year and calculate what the EPS Effect would have been if this had happened. If the bonds had been
converted, there would have been 20,000 additional shares outstanding during the year. There also would
have been more income available to the common shareholders since interest would not have to have been
paid. The interest saved would have been $16,000 (the $200,000 face amount the 8% bond rate - we do not
care about market rate). However, if they had not paid interest, income would have been higher and they
would have had to pay taxes on the $16,000. These taxes would have been $5,440 leaving $10,560 as available
to common shareholders. Dividing this by the 20,000, we get an EPS Effect of $.66. This is less than the basic
EPS so these convertible bonds are dilutive. We add the $10,560 to the net income of $107,000 and the 20,000
shares to the 80,000 outstanding and we get a new calculation of $117,560 divided by 100,000 shares, giving
us a DEPS of $1.18.
C. This answer does not increase income available for common shareholders by the saved interest (net of taxes) from
the bonds that are assumed to have been converted.
D. This answer is incorrect. See the correct answer for a complete explanation.

Question 34 - CIA 1195 IV-32 - Earnings-based Analysis


A company has 100,000 outstanding common shares with a market value of $20 per share. Dividends of $2 per share
were paid in the current year and the company has a dividend payout ratio of 40%. The price to earnings ratio of the
company is
A. 4
B. 2.5
C. 50
D. 10
A. The dividend payout ratio tells us what amount of the company's earnings per share are distributed as
dividends. Since the dividend payout ratio is 40% and the dividend was $2, we can conclude that earnings per
share was $5 ($5 * .4 = $2). The price earnings ratio is calculated as the market price of the stock divided by
the earnings per share. The market price is $20 and the EPS was $5, giving a price earnings ratio of 4.
B. This answer simply divides the earnings per share divided by the dividend per share. See the correct answer for a
complete explanation.
C. This answer is incorrect. See the correct answer for a complete explanation.
D. This is the share price divided by the dividend per share. See the correct answer for a complete explanation.

Question 35 - CMA 1294 2-22 - Short-term Liquidity


The following inventory and sales data are available for the current year for Volpone Company. Volpone uses a
365-day year when computing ratios.
November 30, 2005 November 30,2004

(c) HOCK international, page 22

Part 1 : 11/11/10 07:46:17


Net credit sales
Gross receivables
Inventory
Cost of goods sold

$6,205,000
350,000
960,000
4,380,000

320,000
780,000

Volpone Company's average number of days to collect accounts receivable for the current year is
A. 18.87 days.
B. 19.43 days.
C. 19.71 days.
D. 21.17 days.
A. The average number of days to collect receivables is calculated as 365 divided by the receivables turnover.
Receivables turnover is calculated as credit sales divided by average receivables.
B. The average number of days to collect receivables is calculated as 365 divided by the receivables turnover.
Receivables turnover is calculated as credit sales divided by average receivables.
C. The average number of days to collect receivables is calculated as 365 divided by the receivables turnover.
Receivables turnover is calculated as credit sales divided by average receivables. Credit sales were
$6,205,000 and average receivables were $335,000. This gives a receivables turnover of 18.52. Dividing 365 by
18.52 gives us 19.71 days as the number of days to collect receivables.
D. The average number of days to collect receivables is calculated as 365 divided by the receivables turnover.
Receivables turnover is calculated as credit sales divided by average receivables.

Question 36 - CMA 692 2-26 - Earnings-based Analysis


Information concerning Hamilton's common stock is presented below for the fiscal year ended May 31, 2005.
Common shares outstanding: 750,000
Stated value per share: $15.00
Market price per share: 45.00
Year 1 dividends paid per share: 4.50
Year 2 dividends paid per share: 7.50
Basic earning per share:11.25
Diluted earnings per share: 9.00
The price-earnings ratio for Hamilton's common stock is
A. 3.0 times.
B. 6.0 times.
C. 4.0 times.
D. 5.0 times.
A. The price-earnings ratio is calculated as the current market price divided by the diluted earnings per share. This
answer uses par value instead of diluted earnings per share.
B. The price-earnings ratio is calculated as the current market price divided by the diluted earnings per share. This
answer uses dividends per share instead of diluted earnings per share.
C. The price-earnings ratio is calculated as the current market price divided by the diluted earnings per share. This
answer uses basic earnings per share instead of diluted earnings per share.
D. The price-earnings ratio is calculated as the current market price divided by the diluted earnings per share.
(c) HOCK international, page 23

Part 1 : 11/11/10 07:46:17


The market price is $45 and the diluted earnings per share is $9. This gives a price-earnings ratio of 5.

Question 37 - CMA 688 4-16 - Short-term Liquidity


Which one of the following inventory cost flow assumptions will result in a higher inventory turnover ratio in an
inflationary economy?
A. Weighted average.
B. FIFO.
C. LIFO.
D. Specific identification.
A. The inventory turnover ratio is calculated as cost of goods sold divided by average inventory. Therefore, the
inventory method that gives the lowest value of inventory will give a higher inventory turnover. Weighted average will
give an ending inventory value that is between that of LIFO and FIFO.
B. The inventory turnover ratio is calculated as cost of goods sold divided by average inventory. Therefore, the
inventory method that gives the lowest value of inventory will give a higher inventory turnover. In an inflationary
economy, FIFO provides the highest value for the inventory since it assumes that the oldest (cheapest) units are sold
first and the newest (most expensive) units are in inventory at the end of the period.
C. The inventory turnover ratio is calculated as cost of goods sold divided by average inventory. Therefore,
the inventory method that gives the lowest value of inventory will give a higher inventory turnover. In an
inflationary economy, LIFO provides the lowest value for the inventory since it assumes that the newest (most
expensive) units are sold first and the oldest (cheapest) units are in inventory at the end of the period.
D. The inventory turnover ratio is calculated as cost of goods sold divided by average inventory. Therefore, the
inventory method that gives the lowest value of inventory will give a higher inventory turnover. Even under specific
identification, the inventory turnover will not be higher than LIFO. This is because LIFO assumes that all of the items in
inventory are the oldest whereas specific identification will have some old and some new units in ending inventory.

Question 38 - CMA 679 4-13 - Capital Structure and Solvency


Stock options are frequently provided to officers of companies. Stock options that are exercised improve
A. The total asset turnover.
B. Basic earnings per share.
C. The debt-to-equity ratio.
D. The ownership interest of existing stockholders.
A. Total asset turnover is calculated as sales divided by total assets. When the options are exercised the company will
receive some amount of cash. This will increase total assets and decrease the total asset turnover.
B. When a stock option is exercised there are more shares outstanding. This will decrease the basic earnings per
share because the same amount of income is being divided between a larger number of shares.
C. The debt-to-equity ratio is calculated as total debt divided by total equity. The exercise of the options will
increase total equity and this will decrease the debt-to-equity ratio, which is viewed as an improvement.
D. When a stock option is exercised there are more shares outstanding. This will decrease the ownership interest of
existing shareholders because the number of shares that they own is now a smaller percentage of the larger number
of shares outstanding.
(c) HOCK international, page 24

Part 1 : 11/11/10 07:46:17

Question 39 - CMA 688 4-15 - Short-term Liquidity


The days' sales in receivables ratio will be understated if the company
A. Does not use average receivables in the ratio calculation.
B. Uses average receivables in the ratio calculation.
C. Uses a calendar year for its accounting period.
D. Uses a natural business year for its accounting period.
A. Not using average receivables in the days' sales-in-receivables ratio will not always understate the ratio.
B. The use of average receivables in the days' sales-in-receivables ratio will not always understate the ratio.
C. The use of the calendar year for the accounting period will not always understate the ratio.
D. The days' sales-in-receivables ratio is calculated as the days in the year divided by the receivables turnover
ratio. The receivables turnover ratio is calculated as sales divided by average receivables. In order to
understate the days' sales-in-receivables ratio, either the sales need to be understated or the receivables
turnover needs to be overstated. If the company uses the natural business year for its accounting period, the
accounts receivable balances will be understated. This is because the natural business year will start and end
at the down cycle of the business, understating receivables. Because the receivables will be understated, the
receivables turnover will be higher than it would be otherwise. This will lead to an understated days'
sales-in-receivables ratio.

Question 40 - CMA 693 2-5 - Earnings-based Analysis


Lisa, Inc.
Statement of Financial Position
December 31, 2005
(in thousands)
2005 2004
Assets - Current
Cash
Trading securities
Accounts receivable (net)
Inventories (at lower of cost of market)
Prepaid items
Total Current Assets
Assets - Long-Term
Long-term investments Securities (at lost)
Property, plant & equipment Land (at cost)
Building (net)
Equipment (net)
Intangible assets Patents (net)
Goodwill (net)
Total Long-Term Assets
Total Assets
Liabilities - Current
Notes payable
(c) HOCK international, page 25

$30
20
45
60
15
170

$25
15
30
50
20
140

25
75
80
95
35
20
330
$500

20
75
90
100
17
13
315
$455

$23 $12

Part 1 : 11/11/10 07:46:17


Accounts payable
Accrued interest
Total current liabilities
Liabilities - Long-Term
Notes payable 10% due 12/31/2005
Bonds payable 12% due 12/31/2004
Total long-term debt
Total liabilities
Equity
Preferred - 5% cumulative, $100 par, nonparticipating, authorized, issued and
outstanding, 1,000 shares
Common - $10 par 20,000 shares authorized, 15,000 issued and outstanding shares
Additional paid-in capital - common
Retained earnings
Total Equity
Total Liabilities & Equity

47
15
85

28
15
55

10
15
25
110

10
15
25
80

100 100
150
75
65
390
$500

150
75
50
375
$455

Assuming that there are no preferred stock dividends in arrears, Lisa Inc.'s book value per share of common stock at
December 31, 2005 was
A. $10.00.
B. $18.33.
C. $19.33.
D. $14.50.
A. This answer is incorrect. See the correct answer for a complete explanation.
B. This answer is incorrect. See the correct answer for a complete explanation.
C. The book value per share is calculated as the value of common stockholder's equity divided by the number
of shares outstanding. The common stockholder's equity has a value of $290,000 ($390,000 total
stockholders' equity minus the $100 of preferred shares) and there were 15,000 shares outstanding. This
gives a book value per share of $19.33.
D. This answer is incorrect. See the correct answer for a complete explanation.

Question 41 - CMA 690 4-14 - Short-term Liquidity


Accounts receivable turnover will normally decrease as a result of
A. The write-off of an uncollectible account (assume the use of the allowance for doubtful accounts method).
B. A change in credit policy to lengthen the period for cash discounts.
C. An increase in cash sales in proportion to credit sales.
D. A significant sales volume decrease near the end of the accounting period.
A. Accounts receivable turnover is calculated as annual credit sales divided by the average accounts receivable.
Therefore, this number will decrease if there is a decrease in credit sales or an increase in the average receivables.
Under the allowance for doubtful accounts method, the writing off of a receivable will not effect the accounts
receivable turnover.
B. Accounts receivable turnover is calculated as annual credit sales divided by the average accounts
receivable. Therefore, this number will decrease if there is a decrease in credit sales or an increase in the
average receivables. If the company lengthens the period for cash discounts, more companies will take
longer to pay their bills, which will increase the average receivables. This will, in turn, decrease the accounts
(c) HOCK international, page 26

Part 1 : 11/11/10 07:46:17


receivable turnover.
C. Accounts receivable turnover is calculated as annual credit sales divided by the average accounts receivable.
Therefore, this number will decrease if there is a decrease in credit sales or an increase in the average receivables.
An increase in cash sales by itself will not effect the accounts receivable turnover number.
D. Accounts receivable turnover is calculated as annual credit sales divided by the average accounts receivable.
Therefore, this number will decrease if there is a decrease in credit sales or an increase in the average receivables. A
decrease in credit sales at the end of the period will decrease both the credit sales and the receivables balance at the
end of the year.

Question 42 - CMA 693 2-4 - Short-term Liquidity


Lisa, Inc.
Statement of Financial Position
December 31, 2005
(in thousands)
2005 2004
Assets - Current
Cash
Trading securities
Accounts receivable (net)
Inventories (at lower of cost of market)
Prepaid items
Total Current Assets
Assets - Long-Term
Long-term investments Securities (at lost)
Property, plant & equipment Land (at cost)
Building (net)
Equipment (net)
Intangible assets Patents (net)
Goodwill (net)
Total Long-Term Assets
Total Assets
Liabilities - Current
Notes payable
Accounts payable
Accrued interest
Total current liabilities
Liabilities - Long-Term
Notes payable 10% due 12/31/2005
Bonds payable 12% due 12/31/2004
Total long-term debt
Total liabilities
Equity
Preferred - 5% cumulative, $100 par, nonparticipating, authorized, issued and
outstanding, 1,000 shares
Common - $10 par 20,000 shares authorized, 15,000 issued and outstanding shares
Additional paid-in capital - common
Retained earnings
Total Equity
Total Liabilities & Equity
(c) HOCK international, page 27

$30
20
45
60
15
170

$25
15
30
50
20
140

25
75
80
95
35
20
330
$500

20
75
90
100
17
13
315
$455

$23
47
15
85

$12
28
15
55

10
15
25
110

10
15
25
80

100 100
150
75
65
390
$500

150
75
50
375
$455

Part 1 : 11/11/10 07:46:17


Assume sales and cost of goods sold for 2005 were $300,000 and $220,000, respectively. Lisa Inc.'s inventory
turnover was
A. 5.0 times.
B. 4.4 times.
C. 3.7 times.
D. 4.0 times.
A. This answer is incorrect. See the correct answer for a complete explanation.
B. This answer is incorrect. See the correct answer for a complete explanation.
C. This answer is incorrect. See the correct answer for a complete explanation.
D. The inventory turnover is calculated as average the cost of sales divided by the average annual inventory.
They give us in the problem the annual cost of sales of $220,000. The average inventory is $55,000. The
inventory turnover is calculated as $220,000 divided by $55,000, or 4 times. This means that Lisa sells their
inventory four times a year.

Question 43 - CMA 690 4-20 - Other Analytical Issues


Assume the information below for Ramer Company, for Matson Company, and for their common industry represents a
recent year.
Ramer
Current ratio
3.50
Accounts receivable turnover 5.00
Inventory turnover
6.20
Times interest earned
9.00
Debt-to-equity ratio
0.70
Return on investment
0.15
Dividend payout ratio
0.80
Earnings per share
$3.00

Industry
Matson Average
2.80
3.00
8.10
6.00
8.00
6.10
12.30 10.40
0.40
0.55
0.12
0.15
0.60
0.55
$2.00
--

The attitudes of both Ramer and Matson concerning risk are best explained by the
A. Dividend payout ratio and earnings per share.
B. Debt/equity ratio and times interest earned.
C. Current ratio, accounts receivable turnover, and inventory turnover.
D. Current ratio and earnings per share.
A. These ratios do not give information about the attitudes of the companies concerning risk.
B. These ratios do provide information about the companies' attitudes towards risk. Because Matson is less
leveraged (it has a lower debt-to-equity ratio) and it has a higher times-interest-earned ratio, we know that
Matson is more conservative. Matson is taking on less debt which decreases the chances of defaulting on its
debt and also has less interest payments given its level of income, thereby also reducing its risk.
C. These ratios do not give information about the attitudes of the companies concerning risk.
D. These ratios do not give information about the attitudes of the companies concerning risk.

(c) HOCK international, page 28

Part 1 : 11/11/10 07:46:17

Question 44 - CFM Sample Q. 1 - Earnings-based Analysis


The following information is provided about the common stock of Evergreen Inc. at the end of the fiscal year:
Number of shares outstanding
1,800,000
Par value per share
$10.00
Dividends paid per share (last 12 months)
12.00
Market price per share
108.00
Basic earnings per share
36.00
Diluted earnings per share
24.00
The price/earnings ratio for Evergreen's common stock is
A. 3.0.
B. 10.8.
C. 9.0.
D. 4.5.
A. The P/E ratio is measured as the market price of the share divided by diluted earnings per share (DEPS). This
answer uses basic earnings per share (BEPS) instead of diluted earnings per share.
B. The P/E ratio is measured as the market price of the share divided by diluted earnings per share (DEPS). This
answer uses the par value of the share instead of the diluted earnings per share.
C. The P/E ratio is measured as the market price of the share divided by diluted earnings per share (DEPS). This
answer uses dividends paid rather than diluted earnings per share.
D. The P/E ratio is measured as the market price of the share divided by diluted earnings per share (DEPS).
The market price is $108 and DEPS is $24. This gives a P/E ratio of 4.5 ($108 $24).

Question 45 - CMA 1280 4-7 - Short-term Liquidity


Depoole Company is a manufacturer of industrial products and employs a calendar year for financial reporting
purposes. Assume that total quick assets exceeded total current liabilities both before and after each transaction
described. Further assume that Depoole has positive profits during the year and a credit balance throughout the year
in its retained earnings account.
The early liquidation of a long-term note with cash affects the
A. Current and quick ratio to the same degree.
B. Current ratio but not the quick ratio.
C. Quick ratio to a greater degree than the current ratio.
D. Current ratio to a greater degree than the quick ratio.
A. While both of these ratios will decrease as a result of this transaction, they will not be affected equally. This is
because the numerator in the current and quick ratios are different. The numerator in the current ratio includes
inventory, but the numerator in the quick ratio does not include inventory.
B. Both the current and the quick ratios will be affected by this transaction because this transaction reduces the
current and quick assets used in the calculation of these ratios.
C. The early liquidation of a long-term note with cash will reduce the level of current and quick assets, but will
not affect the level of current liabilities. Therefore, both the current and quick ratios will decrease as a result
(c) HOCK international, page 29

Part 1 : 11/11/10 07:46:17


of this transaction. The reduction in the quick ratio, however, will be greater than the reduction in the current
ratio because the numerator in the quick ratio is smaller. This is because the quick ratio does not include
inventory in the calculation.
D. While both of these ratios will decrease as a result of this transaction, the current ratio will not be affected by a
greater degree than the quick ratio. This is because the numerator in the current and quick ratios are different. The
numerator in the current ratio includes inventory, but the numerator in the quick ratio does not include inventory.
Therefore, the quick ratio will be affected by a greater degree than the current ratio.

Question 46 - CPA 0593 FARE Q60 - Earnings-based Analysis


The following information pertains to Jet Corp.'s outstanding stock for 2005:
Common stock, $5 par value
Shares outstanding, 1/1/05 - 20,000
2-for-1 stock split, 4/1/05 - 20,000
Shares issued, 7/1/05 - 10,000
Preferred stock, $10 par value, 5% cumulative
Shares outstanding, 1/1/05 - 4,000
What are the number of shares Jet should use to calculate 2005 earnings per share?
A. 45,000
B. 40,000
C. 54,000
D. 50,000
A. In order to calculate the number of shares to be used in the EPS calculation, we need to treat shares
issued as part of a stock split as having been outstanding for the entire period. This means that the 20,000
shares issued on April 1 will be treated as outstanding the whole year. This gives the company 40,000
weighted average shares. The shares that were issued on July 1 were outstanding for only 1/2 of the year so
they are multiplied by 1/2 in order to calculate the weighted average number of shares. This is 5,000 and when
added to the other 40,000 shares outstanding, gives the company 45,000 weighted average shares
outstanding for 2005.
B. This answer fails to take into account the shares that were issued on July 1. See the correct answer for a complete
explanation.
C. This answer incorrectly includes the preferred shares. They are not included in the EPS calculation. This answer
also incorrectly treats the shares that were issued on July 1 as if they had been outstanding for the entire year. See
the correct answer for a complete explanation.
D. This answer incorrectly treats the shares that were issued on July 1 as if they had been outstanding for the entire
year. See the correct answer for a complete explanation.

Question 47 - CMA 687 4-27 - Capital Structure and Solvency


When compared to a debt t -assets ratio, a debt to equity ratio would
A. Be higher than the debt to assets ratio.
B. Have no relationship at all to the debt to assets ratio.
(c) HOCK international, page 30

Part 1 : 11/11/10 07:46:17


C. Be lower than the debt to assets ratio.
D. Be about the same as the debt to assets ratio.
A. Because the asset balance of a company is higher than its equity, a debt-to-equity ratio will be higher than
a debt-to-asset ratio. This is because when equity is used in the denominator, the denominator will be smaller
than when assets are used in the denominator. We know that the assets of the company must be higher than
the equity of the company because of the accounting equation of assets = liabilities + equity.
B. The debt-to-equity ratio will be higher than the debt-to-assets ratio. See the correct answer for a complete
explanation.
C. The debt-to-equity ratio will be higher than the debt-to-assets ratio. See the correct answer for a complete
explanation.
D. These two ratios could not be the same because the assets and equity of a company will be different from each
other (except in the unusual situation in which the company has no debt).

Question 48 - CMA 1291 P2 Q20 - Earnings-based Analysis


Sands, Inc. uses a calendar year for financial reporting. The company is authorized to issue 5,000,000 shares of $10
par common stock. At no time has Sands issued any potentially dilutive securities. Listed below is a summary of
Sands' common stock activities.
Number of common shares issued and outstanding at Dec. 31, 2003:1,000,000
Shares issued as a result of a 10% stock dividend on Sept. 30, 2004: 100,000
Shares issued for cash on March 31, 2005:
1,000,000
Number of common shares issued and outstanding at Dec. 31, 2005:2,100,000
A two-for-one stock split of Sands' common stock took place on March 31, 2006.
The weighted-average number of common shares used in computing earnings per common share for 2005 on the
2005 comparative income statement was
A. 3,700,000.
B. 1,600,000.
C. 1,850,000.
D. 2,100,000.
A. This answer includes the effect of the 2-for-1 stock split that occurred on March 31, 2006. However, the question
asks for the weighted-average number of common shares for 2005 on the 2005 comparative income statement. The
stock split did not take place until after the 2005 comparative income statement was issued.
B. This answer incorrectly assumes that the shares issued on March 31, 2005 were outstanding for only 6 months of
the year, instead of 9 months.
C. At the end of 2004 there were 1,100,000 shares outstanding. To this, the company needs to add the effect
of any 2005 share transactions. The only transaction was the issuance of 1,000,000 shares on March 31. Since
these shares were outstanding for only 9 months of the year, they will count as only 75% for the
weighted-average share calculation. These shares will become 750,000 shares for this calculation, giving the
company 1,850,000 shares outstanding for the 2005 EPS calculation.
D. This answer assumes that the shares that were issued in 2005 were outstanding for the entire period. In fact, they
were outstanding for only 9 months and therefore should be considered to be only 750,000 (1,000,000 * 9/12) shares

(c) HOCK international, page 31

Part 1 : 11/11/10 07:46:17


for the purposes of the weighted-average outstanding calculation.

Question 49 - CMA 685 4-17 - Capital Structure and Solvency


If the ratio of total liabilities to equity increases, a ratio that must also increase is
A. The current ratio.
B. Return on equity.
C. Times interest earned.
D. Total liabilities to total assets.
A. In order to determine the effect on the current ratio we would need to know the change in current liabilities and
current assets, neither of which are known from the information given.
B. In order to calculate the return on equity, we would need to know the level of income of the company.
C. The times interest earned ratio is not automatically affected if the ratio of liabilities to equity increases. We would
need to know the levels of income and interest expense to make a conclusion about the times interest earned ratio.
D. If total liabilities increase the ratio of total liabilities to total assets must also increase.

Question 50 - CMA 690 4-13 - Short-term Liquidity


To determine the operating cycle for a retail department store, which one of the following pairs of items is needed?
A. Asset turnover and return on sales.
B. Cash turnover and net sales.
C. Days' sales in accounts receivable and average merchandise inventory.
D. Accounts receivable turnover and inventory turnover.
A. The operating cycle is the length of time it takes to convert an investment of cash in inventory back into cash
(through collections of sales). It is calculated as the days sales in inventory + days sales in receivables. This
information will not allow us to calculate the operating cycle.
B. The operating cycle is the length of time it takes to convert an investment of cash in inventory back into cash
(through collections of sales). It is calculated as the days sales in inventory + days sales in receivables. This
information will not allow us to calculate the operating cycle.
C. The operating cycle is the length of time it takes to convert an investment of cash in inventory back into cash
(through collections of sales). It is calculated as the days sales in inventory + days sales in receivables. In addition to
the average merchandise inventory, we also need to know the daily cost of goods sold in order to calculate the days
sales in inventory.
D. The operating cycle is the length of time it takes to convert an investment of cash in inventory back into
cash (through collections of sales). It is calculated as the days sales in inventory + days sales in receivables.
Both of these amounts can be calculated from the information provided in this choice.

Question 51 - CMA 1291 P2 Q22 - Earnings-based Analysis

(c) HOCK international, page 32

Part 1 : 11/11/10 07:46:17


Sands, Inc. uses a calendar year for financial reporting. The company is authorized to issue 5,000,000 shares of $10
par common stock. At no time has Sands issued any potentially dilutive securities. Listed below is a summary of
Sands' common stock activities.
Number of common shares issued and outstanding at Dec. 31, 2003:1,000,000
Shares issued as a result of a 10% stock dividend on Sept. 30, 2004: 100,000
Shares issued for cash on March 31, 2005:
1,000,000
Number of common shares issued and outstanding at Dec. 31, 2005:2,100,000
A two-for-one stock split of Sands' common stock took place on March 31, 2006.
The weighted-average number of common shares to be used in computing earnings per common share for 2006 on
the 2006 comparative income statement is
A. 3,675,000.
B. 4,200,000.
C. 3,150,000.
D. 2,100,000.
A.
This answer assumes that the shares issued in the stock split were outstanding for nine months. However, shares
issued in a stock split or as a stock dividend are considered to have been outstanding from January 1 of the first year
presented.

B.
At the beginning of 2006 there were 2,100,000 shares outstanding. In 2006, there was a 2-for-1 stock split and
the shares that were issued as part of the stock split are considered to have been outstanding for the entire
year. Therefore, the 2006 weighted-average shares outstanding on the 2006 comparative income statement is
4,200,000.
C. This answer assumes that the shares issued in the stock split were outstanding for six months. However, shares
issued in a stock split or as a stock dividend are considered to have been outstanding from January 1 of the first year
presented.
D. This answer does not take into account the 2-for-1 stock split that occurred in 2006.

Question 52 - CMA 1280 4-6 - Short-term Liquidity


Depoole Company is a manufacturer of industrial products and employs a calendar year for financial reporting
purposes. Assume that total quick assets exceeded total current liabilities both before and after each transaction
described. Further assume that Depoole has positive profits during the year and a credit balance throughout the year
in its retained earnings account.
The issuance of serial bonds in exchange for an office building, with the first installment of the bonds due late this year,
A. Affects all of the answers as indicated.
B. Decreases the quick ratio.
C. Decreases net working capital.
D. Decreases the current ratio.

(c) HOCK international, page 33

Part 1 : 11/11/10 07:46:17


A. All of the items listed in the other options will be affected in the manner in the choices. This makes this
choice the correct answer. Please see the other choices for complete explanations of the individuals items.
B. Because the first installment of the bonds is due later this year, this will increase the current liabilities, thereby
decreasing the quick ratio. Because the bonds were issued in exchange for an office building, they did not receive any
cash - meaning that there was no increase in current assets. While it is correct that the quick ratio does decrease, this
answer is not correct because there is a better choice available.
C. Because the first installment of the bonds is due later this year, this will increase the current liabilities, thereby
decreasing net working capital. Because the bonds were issued in exchange for an office building, they did not receive
any cash - meaning that there was no increase in current assets. While it is correct that net working capital does
decrease, this answer is not correct because there is a better choice available.
D. Because the first installment of the bonds is due later this year, this will increase the current liabilities, thereby
decreasing the current ratio. Because the bonds were issued in exchange for an office building, they did not receive
any cash - meaning that there was no increase in current assets. While it is correct that the current ratio does
decrease, this answer is not correct because there is a better choice available.

Question 53 - CMA 688 4-14 - Short-term Liquidity


A high sales-to-working-capital ratio could indicate
A. The firm is not susceptible to liquidity problems.
B. The firm is undercapitalized.
C. Sales are not adequate relative to available working capital.
D. Unprofitable use of working capital.
A. The high level of sales would indicate that the company also has large payments to suppliers and other costs of
operations. The low level of working capital may make it difficult at times for the company to meet those liabilities as
they come due.
B. Because the company is generating such a high return on the working capital, it may indicate that the
company does not have enough capital to invest in what it could invest in. This is not necessarily the case,
but a possible situation of the company. The company may be able to utilize additional capital to further
increase sales, to reduce the risk related to the high sales-to-working-capital ratio, or for other purposes.
C. Because the company has high levels of sales for its working capital, this indicates that the sales are adequate
given the level of working capital.
D. Because the company has high levels of sales for its working capital, this indicates that the working capital is being
used effectively.

Question 54 - CIA 590 IV-47 - Short-term Liquidity


Given an acid test ratio of 2.0, current assets of $5,000, and inventory of $2,000, the value of current liabilities is
A. $1,500
B. $3,500
C. $2,500
D. $6,000
A. The acid test ratio is calculated as follows: (cash + receivables) / current liabilities. Inventory a current
(c) HOCK international, page 34

Part 1 : 11/11/10 07:46:17


asset, but it is not included in the acid test ratio calculation. This means that for this calculation the
numerator is only $3,000. Since the ratio is 2, the current liabilities must be $1,500.
B. This answer adds the value of inventory to the current assets figure instead of subtracting it.
C. This answer incorrectly includes the value of the inventory in the numerator of the calculation. Inventory is included
in the current ratio, but not the acid test ratio.
D. This answer multiplies the numerator by 2 instead of dividing it by 2.

Question 55 - CMA 1285 4-23 - Short-term Liquidity


Windham Company has current assets of $400,000 and current liabilities of $500,000. Windham Company's current
ratio would be increased by
A. Refinancing a $100,000 long-term loan with short-term debt.
B. The purchase of $100,000 of inventory on account.
C. The payment of $100,000 of accounts payable.
D. The collection of $100,000 of accounts receivable.
A. Currently, the current ratio is .8. This transaction would increase current liabilities to $600,000 and current assets
would remain unchanged. After this, the current ratio would be .667, a decrease over what it was before this
transaction.
B. Currently, the current ratio is .8. This transaction would increase current assets to $500,000 and current
liabilities to $600,000. After this, the current ratio would be .833, an increase over what it was before this
transaction.
C. Currently, the current ratio is .8. This transaction would decrease current assets to $300,000 and current liabilities
to $400,000. After this, the current ratio would be .75, a decrease over what it was before this transaction.
D. This transaction would have no impact on the current ratio because all that is happening is the current asset of
accounts receivable is being converted into the current asset of cash.

Question 56 - CMA 685 4-16 - Earnings-based Analysis


A drop in the market price of a firm's common stock will immediately affect its
A. Dividend yield.
B. Debt to net worth ratio.
C. Dividend payout ratio.
D. Return on equity.
A. The dividend yield is calculated as the dividend per common share divided by the market price of the
share. Therefore, a change in the market price will change the dividend yield.
B. The debt to net worth ratio is not calculated using the share's market price.
C. The dividend payout ratio is not calculated using the share's market price.
D. The return on equity is not calculated using the market price of the firm's stock.

(c) HOCK international, page 35

Part 1 : 11/11/10 07:46:17

Question 57 - CMA 688 4-2 - Capital Structure and Solvency


The data presented below shows actual figures for selected accounts of McKeon Company for the fiscal year ended
May 31, 2004, and selected budget figures for the 2005 fiscal year. McKeon's controller is in the process of reviewing
the 2005 budget and calculating some key ratios based on the budget. McKeon Company monitors yield or return
ratios using the average financial position of the company. (Round all calculations to three decimal places if
necessary.)

Current assets
Noncurrent assets
Current liabilities
Long-term debt
Common stock ($30 par value)
Retained earnings

5/31/05
$210,000
275,000
78,000
75,000
300,000
32,000

2005 Operations
Sales*
Cost of goods sold
Interest expense
Income taxes (40% rate)
Dividends declared and paid in 2005
Administrative expense
*All sales are credit sales.

5/31/04
$180,000
255,000
85,000
30,000
300,000
20,000

$350,000
160,000
3,000
48,000
60,000
67,000

Composition of Current Assets

Cash
Accounts receivable
Inventory
Other

5/31/05
$20,000
100,000
70,000
20,000
$210,000

5/31/04
$10,000
70,000
80,000
20,000
$180,000

McKeon Company's debt to total asset ratio for 2005 is


A. 0.237
B. 0.315
C. 0.264
D. 0.352
A. This answer is incorrect. See the correct answer for a full explanation.
B. The debt-to-total asset ratio is all debt (current and non-current) divided by total assets. The budgeted debt
for 2005 is equal to $153,000 ($75,000 + $78,000) and the total assets are $485,000 ($210,000 + $275,000). Thus,
the ratio is .315.
C. This is the debt-to-total asst ratio for 2004.
D. This answer is incorrect. See the correct answer for a full explanation.

(c) HOCK international, page 36

Part 1 : 11/11/10 07:46:17


Question 58 - CMA 691 2-7 - Capital Structure and Solvency
Selected data from Ostrander Corporation's financial statements for the years indicated are presented in thousands.
2005 Operations
Net sales
Cost of goods sold
Interest expense
Income tax
Gain on disposal of a segment (net of tax)
Net income

$4,175
2,880
50
120
210
385

December 31, 2005

Cash
Trading securities
Accounts receivable (net)
Merchandise inventory
Tangible fixed assets
Total assets
Current liabilities
Total liabilities
Common stock outstanding
Retained earnings

2005
$32
169
210
440
480
1,397
370
790
226
381

2004
$28
172
204
420
440
1,320
368
750
210
360

The times interest was earned for Ostrander Corporation for 2005 is
A. 7.70 times.
B. 3.50 times.
C. 4.50 times.
D. 6.90 times.
A. This answer uses net income to calculate the times interest earned and does not adjust it for interest, taxes and
nonrecurring items.
B. This answer adjusts net income only for the nonrecurring item and does not adjust for interest and taxes.
C. This answer is incorrect. See the correct answer for a complete explanation.
D. Times interest earned is calculated as the operating income before interest and taxes divided by the
interest expense. The operating income before interest and taxes is $345 ($385 income plus the $50 of
interest and $120 of taxes minus the $210 gain from the disposal since it is not operations). Interest was $50,
and this gives a times interest earned of 6.90.

Question 59 - CIA 0594 P4 Q31 - Earnings-based Analysis


A company has net income for the current year of $120,000 and pays $5,000 in dividends to its preferred
shareholders and $20,000 in dividends to its common shareholders. The weighted average number of common
shares outstanding for the year is 1,500, and the weighted average number of preferred shares outstanding for the
year is 2,500. Earnings per share for this company for the current year, to the nearest cent, is
A. $76.67
(c) HOCK international, page 37

Part 1 : 11/11/10 07:46:17


B. $40.00
C. $66.67
D. $60.00
A. Earnings per share is calculated as the income available to common shareholders divided by the weighted
average number of common shares outstanding (which is given to us in the problem as 1,500). The income
available to common shareholders is equal to net income minus preferred dividends. Net income was
$120,000 and the preferred dividends were $5,000, giving an income available to common shareholders of
$115,000. (Common dividends are not adjusted for because the common dividends are still available to the
common shareholders.) $115,000 divided by 1,500 shares gives an EPS of $76.67.
B. This answer incorrectly subtracts common dividends from net income instead of preferred dividends and then
divides this number by the number of preferred shares instead of common shares outstanding.
C. This answer is incorrect. See the correct answer for a complete explanation.
D. This answer is incorrect. See the correct answer for a complete explanation.

Question 60 - CMA 1293 2-13 - Short-term Liquidity


In computing inventory turnover, the preferred base to use is the
A. Sales base because it provides turnover rates that are considerably higher.
B. Cost of sales base because it eliminates any changes due solely to sales price changes.
C. Sales base because it more clearly represents operational activity.
D. Sales base because it is more likely to reflect a change in trend.
A. Inventory turnover is calculated as cost of sales divided by the average inventory for the period. If sales were used
instead of cost of sales, this ratio would change simply as a result of the company changing the price that they sell the
product for. Therefore, sales is not a good base to use.
B. Inventory turnover is calculated as cost of sales divided by the average inventory for the period. If sales
were used instead of cost of sales, this ratio would change simply as a result of the company changing the
price that they sell the product for. Therefore, using cost of sales instead insulates this ratio from a change in
the selling price of the product.
C. Inventory turnover is calculated as cost of sales divided by the average inventory for the period. If sales were used
instead of cost of sales, this ratio would change simply as a result of the company changing the price that they sell the
product for. Therefore, sales is not a good base to use.
D. Inventory turnover is calculated as cost of sales divided by the average inventory for the period. If sales were used
instead of cost of sales, this ratio would change simply as a result of the company changing the price that they sell the
product for. Therefore, sales is not a good base to use.

Question 61 - CMA 690 4-12 - Short-term Liquidity


Merit, Inc. uses the direct write-off method to account for uncollectible accounts receivable. If the company
subsequently collects an account receivable that was written off in a prior accounting period, the effect of the
collection of the account receivable on Merit's current ratio and total working capital would be
A. Both items will increase.
B. Neither item will change.
(c) HOCK international, page 38

Part 1 : 11/11/10 07:46:17


C. The current ratio will increase, but the working capital will be unchanged.
D. The current ratio will be unchanged, but working capital will increase.
A. Under the direct write-off method, the collection of a previously written off receivable involves a debit
(increase) to cash and a credit (increase) to retained earnings. This transaction therefore increase current
assets, but leaves current liabilities unchanged. Therefore, both the current ratio and working capital will
increase.
B. Under the direct write-off method, the collection of a previously written off receivable involves a debit (increase) to
cash and a credit (increase) to retained earnings. This transaction therefore increase current assets, but leaves
current liabilities unchanged. Therefore, both the current ratio and working capital will increase.
C. Under the direct write-off method, the collection of a previously written off receivable involves a debit (increase) to
cash and a credit (increase) to retained earnings. This transaction therefore increase current assets, but leaves
current liabilities unchanged. Therefore, both the current ratio and working capital will increase.
D. Under the direct write-off method, the collection of a previously written off receivable involves a debit (increase) to
cash and a credit (increase) to retained earnings. This transaction therefore increase current assets, but leaves
current liabilities unchanged. Therefore, both the current ratio and working capital will increase.

Question 62 - CMA 688 4-17 - Other Analytical Issues


In financial statement analysis, expressing all financial statement items as a percentage of base-year figures is called
A. Horizontal common size analysis.
B. Ratio analysis.
C. Vertical common size analysis.
D. Trend analysis.
A. Horizontal common-size analysis occurs when the financial information is presented as a percentage of
the some company's financial information from a previous period. The current year amounts are stated in
comparison to the base period for that company, which is given a value of 100%.
B. Ratio analysis is the term that is used to encompass many different elements of financial analysis. It is looking at
the relationships between numbers.
C. Vertical, common-size analysis creates financial statements in which each component is measured as a
percentage of another element of the financial statements for that same period. For example, all items on the balance
sheet are measured as a percentage of total assets and all income statement items are measured as a percentage of
total sales.
D. Trend analysis is the process of looking at past data and attempting to determine what future results for that data
will be.

Question 63 - CPA 0592 FARE1 Q60b - Earnings-based Analysis


On June 30, 2004, Lomond, Inc. issued twenty, $10,000, 7% bonds at par. Each bond was convertible into 200 shares
of common stock. On January 1, 2005, 10,000 shares of common stock were outstanding. None of the bonds were
converted during the period. The following amounts were reported in Lomond's income statement for the year ended
December 31, 2005:
Revenues

$977,000
(c) HOCK international, page 39

Part 1 : 11/11/10 07:46:17


Operating expenses
$(920,000)
Interest on bonds
$(14,000)
Income before income tax
$43,000
Income tax at 30%
$(12,900)
Net income
$30,100
What amount should Lomond report as its 2005 diluted earnings per share?
A. $2.92
B. $2.85
C. $2.50
D. $3.50
A. This answer is incorrect. See the correct answer for a complete explanation.
B. In order to calculate DEPS, we need to assume that the bonds had been converted at the beginning of the
year and calculate what the EPS Effect would have been if this had happened. If the bonds had been
converted, there would have been 4,000 additional shares outstanding during the year. There also would have
been more income available to the common shareholders since interest would not have to have been paid.
The interest saved would have been $14,000 (the $200,000 face amount the 7% bond rate). However, if they
had not paid interest, income would have been higher and they would have had to pay taxes on the $14,000.
These taxes would have been $4,200 leaving $9,800 as available to common shareholders. Dividing this by the
4,000, we get an EPS Effect of $2.45. This is less than the basic EPS so these convertible bonds are dilutive.
We add the $9,800 to the net income of $30,100 and the 4,000 shares to the 10,000 outstanding and we get a
new calculation of $39,900 divided by 14,000 shares, giving us a DEPS of $2.85.
C. This answer is incorrect. See the correct answer for a complete explanation.
D. This answer is incorrect. See the correct answer for a complete explanation.

Question 64 - CPA 1190 FARE Q52 - Earnings-based Analysis


Poe Co. had 300,000 shares of common stock issued and outstanding at December 31, 2004. No common stock was
issued during 2005. On January 1, 2005 Poe issued 200,000 shares of nonconvertible preferred stock. During 2005,
Poe declared and paid $75,000 cash dividends on the common stock and $60,000 on the preferred stock. Net income
for the year ended December 31, 2005 was $330,000. What should be Poe's 2005 earnings per common share?
A. $0.65
B. $0.90
C. $1.10
D. $0.85
A. This answer incorrectly deducts the common dividend from net income to calculate the income available to
common shareholders. The preferred dividend should be subtracted, but the common dividend should not be
subtracted because the money of the common dividend is still available to the common shareholder.
B. Poe has no potential common stock, so its EPS will be its Basic EPS. EPS is calculated as the income
available to common shareholders divided by the weighted average number of common shares outstanding.
The income available to common shareholders is the net income ($330,000) minus the dividends paid on
preferred shares ($60,000), or $270,000. The dividends paid on the common shares are not subtracted
because the common dividends are still available to common shareholders. The weighted average number of
shares outstanding for the period was 300,000. Dividing the income available by the common shares
outstanding, we get an EPS of $.90.

(c) HOCK international, page 40

Part 1 : 11/11/10 07:46:17


C. This answer fails to take into account the preferred dividend. This needs to be subtracted from net income to
calculate the income available to common shareholders. See the correct answer for a complete explanation.
D. This answer is incorrect. See the correct answer for a complete explanation.

Question 65 - CMA 695 2-3 - Short-term Liquidity


CPZ Enterprises had the following account information.
Accounts receivable
Accounts payable
Bonds payable, due in 10 years
Cash
Interest payable, due in three months
Inventory
Land
Notes payable, due in six months
Prepaid expences

$200,000
80,000
300,000
100,000
10,000
400,000
250,000
50,000
40,000

The company has an operating cycle of five months.


What will happen to the current and quick ratios if CPZ Enterprises uses cash to pay 50 percent of the accounts
payable?
A. Both ratios will increase.
B. The current ratio will increase and the quick ratio will decrease.
C. The current ratio will decrease and the quick ratio will increase.
D. Both ratios will decrease.
A. The current ratio is calculated as current assets divided by current liabilities. Current assets include
accounts receivable ($200,000), cash ($100,000), inventory ($400,000) and prepaid expenses ($40,000). Current
liabilities include accounts payable ($80,000), interest payable ($10,000) and notes payable ($50,000). Note
that even though the notes payable are due after the operating cycle is over, the distinction for a current asset
or liability is that it will be converted or settled within 12 months or the operating cycle, whichever is longer.
The total current assets are $740,000 and current liabilities are $140,000. This gives us a current ratio of 5.29
before the payment is made. If the payment of the payables is made, the total current assets will become
$700,000 and the current liabilities will become $100,000. This gives a new current ratio of 7, an increase in
the current ratio. The quick ratio is calculated as quick assets (current assets excluding inventory and
prepaid expenses) divided by current liabilities. Quick assets include accounts receivable ($200,000) and cash
($100,000). Current liabilities include accounts payable ($80,000), interest payable ($10,000) and notes payable
($50,000). Note that even though the notes payable are due after the operating cycle is over, the distinction for
a current asset or liability is that it will be converted or settled within 12 months or the operating cycle,
whichever is longer. The total quick assets are $300,000 and current liabilities are $140,000. This gives us a
quick ratio of 2.14 before the payment of the payables. If the payment of the payables is made, the total quick
assets will become $260,000 and the current liabilities will become $100,000. This gives a new current ratio of
2.6, an increase in the current ratio.
B. This answer is incorrect. See the correct answer for a complete explanation.
C. This answer is incorrect. See the correct answer for a complete explanation.
D. This answer is incorrect. See the correct answer for a complete explanation.

(c) HOCK international, page 41

Part 1 : 11/11/10 07:46:17

Question 66 - CMA 690 4-11 - Short-term Liquidity


Rice, Inc. uses the allowance method to account for uncollectible accounts. An account receivable that was previously
determined uncollectible and written off was collected during May. The effect of the collection on Rice's current ratio
and total working capital is as follows:
A. The current ratio will be unchanged, but working capital will increase.
B. Both ratios will decrease.
C. There is no impact on either of the ratios.
D. Both ratios will increase.
A. When a receivable that was previously written off is collected under the allowance method, the journal entry to
record this is a debit (increase) to cash and a credit (increase) to the allowance for doubtful debt account. The
allowance for doubtful debts is a contra asset account, which means that it functions as a liability, but is recorded as a
current asset, reducing the values of the current assets. This transaction then increases and decreases current assets
by the same amount, leading to no change in the current ratio or working capital.
B. When a receivable that was previously written off is collected under the allowance method, the journal entry to
record this is a debit (increase) to cash and a credit (increase) to the allowance for doubtful debt account. The
allowance for doubtful debts is a contra asset account, which means that it functions as a liability, but is recorded as a
current asset, reducing the values of the current assets. This transaction then increases and decreases current assets
by the same amount, leading to no change in the current ratio or working capital.
C. When a receivable that was previously written off is collected under the allowance method, the journal
entry to record this is a debit (increase) to cash and a credit (increase) to the allowance for doubtful debt
account. The allowance for doubtful debts is a contra asset account, which means that it functions as a
liability, but is recorded as a current asset, reducing the values of the current assets. This transaction then
increases and decreases current assets by the same amount, leading to no change in the current ratio or
working capital.
D. When a receivable that was previously written off is collected under the allowance method, the journal entry to
record this is a debit (increase) to cash and a credit (increase) to the allowance for doubtful debt account. The
allowance for doubtful debts is a contra asset account, which means that it functions as a liability, but is recorded as a
current asset, reducing the values of the current assets. This transaction then increases and decreases current assets
by the same amount, leading to no change in the current ratio or working capital.

Question 67 - CMA 696 1-15 - Other Analytical Issues


Spotech Co.'s budgeted sales and budgeted cost of sales for the coming year are $212,000,000 and $132,500,000,
respectively. Short-term interest rates are expected to average 5%. If Spotech could increase inventory turnover from
its current 8 times per year to 10 times per year, its expected cost savings in the current year would be
A. $165,625
B. $250,000
C. $331,250
D. $82,812
A.
If Spotech is able to turn their inventory over more frequently, they will have a lower average inventory and
they will be able to invest the money that is no longer invested in inventory at 5%. In order to answer this
question, we need to calculate what the average inventory is with an inventory turnover of 8 times and what it
(c) HOCK international, page 42

Part 1 : 11/11/10 07:46:17


would be if the inventory turnover were 10 times instead. Then, we can calculate how much the company
could earn by investing the difference for one year at 5%.
Inventory turnover is calculated as cost of sales divided by average inventory. Therefore, we can calculate
average inventory by dividing cost of sales by the inventory turnover. Currently, average inventory is
$16,562,500 ($132,500,000 8). If inventory turnover increases to 10 times, the average inventory will be
$13,250,000 ($132,500,000 10). This difference of $3,312,500 will be able to be invested at 5%, earning a total
of $165,625.
B. This answer is incorrect. See the correct answer for a complete explanation.
C. If Spotech is able to turn their inventory over more frequently, they will have a lower average inventory and they will
be able to invest the money that is no longer invested in inventory at 5%. This answer uses an investment rate of 10%
instead of 5%.
D. If Spotech is able to turn their inventory over more frequently, they will have a lower average inventory and they will
be able to invest the money that is no longer invested in inventory at 5%. This answer is 0.5% of the amount that is
currently invested in inventory on average. See the correct answer for a complete explanation.

Question 68 - CMA 1295 2-13 - Short-term Liquidity


All of the following financial indicators are measures of either liquidity or activity except the
A. Average collection period in days.
B. Merchandise inventory turnover.
C. Accounts receivable turnover.
D. Times-interest-earned ratio.
A. Liquidity ratios have to do with the ability of the company to pay its short-term liabilities as the become due. Activity
ratios measure the ability of the company to use its assets to generate sales and profits. The average collection period
is a measure of activity.
B. Liquidity ratios have to do with the ability of the company to pay its short-term liabilities as the become due. Activity
ratios measure the ability of the company to use its assets to generate sales and profits. The merchandise inventory
turnover is a measure of activity.
C. Liquidity ratios have to do with the ability of the company to pay its short-term liabilities as the become due. Activity
ratios measure the ability of the company to use its assets to generate sales and profits. The accounts receivable
turnover is a measure of activity.
D. . Liquidity ratios have to do with the ability of the company to pay its short-term liabilities as the become
due. Activity ratios measure the ability of the company to use its assets to generate sales and profits. Times
interest earned measures neither the ability to meet current liabilities or the use of assets to generate
revenues or profits. 64 deleted - not relevant

Question 69 - CPA Released 2000 - Earnings-based Analysis


Deck Co. had 120,000 shares of common stock outstanding at January 1, 2005. On July 1, 2005, it issued 40,000
additional shares of common stock. Outstanding all year were 10,000 shares of nonconvertible cumulative preferred
stock. What is the number of shares that Deck should use to calculate 2005 earnings per share?
A. 150,000
(c) HOCK international, page 43

Part 1 : 11/11/10 07:46:17


B. 140,000
C. 160,000
D. 170,000
A. This answer incorrectly includes the preferred shares. Preferred shares are not included in the EPS calculation.
See the correct answer for a complete explanation.
B. To calculate the number of shares to use in the EPS calculation we start with the 120,000 shares that were
outstanding at the beginning of the year and add to that the effect of the share transactions during the year.
The shares that were issued on July 1 were outstanding for only 1/2 of the year so they are multiplied by 1/2 in
order to calculate the weighted average number of shares. This is 20,000 and when added to the other
120,000 shares outstanding, gives the company 140,000 shares outstanding for 2005. The preferred
nonconvertible shares are not relevant because they are not convertible. These preferred shares would not be
included in any Diluted EPS calculation.
C. This answer incorrectly treats the shares that were issued on July 1 as having been outstanding for the entire
period. See the correct answer for a complete explanation.
D. This answer incorrectly treats the shares that were issued on July 1 as having been outstanding for the entire
period. This answer also incorrectly includes the preferred shares. Preferred shares are not included in the EPS
calculation. See the correct answer for a complete explanation.

Question 70 - CMA 685 4-13 - Earnings-based Analysis


Which one of the following statements about the price-earnings (P-E) ratio is correct?
A. A company with high growth opportunities ordinarily has a high P-E ratio.
B. A P-E ratio has more meaning when a firm has losses than when it has profits.
C. A P-E ratio expresses the relationship between a firm's market price and its net sales.
D. A P-E ratio has more meaning when a firm has abnormally low profits in relation to its asset base.
A. The P-E ratio is measured as the market price of the shares divided by the diluted earnings per share. If the
price of the share is many times higher than the diluted EPS of a share, the market is expecting that there will
be a much greater return in the future. This is because the current earnings of the share do not support such
a high value so there must be an expectation that the future will be better than the present for the company.
B. The P-E ratio has less meaning when the firm has losses than when it has profits.
C. The P-E ratio is measured as the market price of the shares divided by the diluted earnings per share and
expresses the relationship between the price of a share and the earnings of that share.
D. The P-E ratio has less meaning when the firm has low profits.

Question 71 - CMA 692 2-27 - Short-term Liquidity


If a company decided to change from the first-in, first-out (FIFO) inventory method to the last-in, first-out (LIFO)
method during a period of rising prices, its
A. Debt-to-equity ratio would be decreased.
B. Inventory turnover ratio would be reduced.
C. Current ratio would be reduced.
D. Cash flow would be decreased.
(c) HOCK international, page 44

Part 1 : 11/11/10 07:46:17

A. The debt-to-equity ratio is not directly affected by the method of inventory tracking that is used. However, it is
affected through a higher or lower profit from the sale of inventory. If the company switched to LIFO, they would have
a higher cost of goods sold and this will decrease profits. The lower profits will decrease equity, which would increase
the debt-to-equity ratio.
B. If the company changes from FIFO to LIFO during a period of rising prices, their ending inventory value will
decrease. This is because under LIFO the ending inventory is made up of the oldest items in inventory and these are
cheaper in a period of rising prices. Also, the cost of goods sold will increase because the company is now selling the
newer, more expensive items in inventory. Inventory turnover is calculated as cost of goods sold divided by the
average inventory. Since this change to LIFO would increase the numerator and decrease the denominator, this ratio
will increase as a result of the change.
C. If the company changes from FIFO to LIFO during a period of rising prices, their ending inventory value will
decrease. This is because under LIFO the ending inventory is made up of the oldest items in inventory and
these are cheaper in a period of rising prices. Also, the cost of goods sold will increase because the company
is now selling the newer, more expensive items in inventory. Since inventory is part of the numerator of the
current ratio and it is now smaller, the current ratio for the company would be reduced if it switched from
FIFO to LIFO.
D. The cash flows of the company are not affected by the method of inventory tracking.

Question 72 - CMA 1293 2-17 - Short-term Liquidity


Norton, Inc. has a 2-to-1 current ratio. This ratio would increase to more than 2 to 1 if
A. A previously declared stock dividend were distributed.
B. The company wrote off an uncollectible receivable.
C. The company sold merchandise on open account that earned a normal gross margin.
D. The company purchased inventory on open account.
A. A distribution of a stock dividend does not affect current assets or current liabilities, and therefore the current ratio
would remain unchanged.
B. Writing off a receivable will decrease both the receivables account and the related allowance for bad debts.
Therefore, net receivables will remain unchanged and the current ratio will remain unchanged.
C. When the company sells inventory at a profit, the current assets will increase. This is because the
receivable that is created is larger than the inventory that was written off. This increase in current assets will
increase the current ratio.
D. When a company purchase inventory on account the current assets and the current liabilities of the company will
both increase. Since the company currently has a current ratio of 2, this equal increase in the numerator and
denominator will actually decrease the current ratio of the company.

Question 73 - CMA 691 2-8 - Capital Structure and Solvency


Selected data from Ostrander Corporation's financial statements for the years indicated are presented in thousands.
2005 Operations
Net sales

$4,175
(c) HOCK international, page 45

Part 1 : 11/11/10 07:46:17


Cost of goods sold
Interest expense
Income tax
Gain on disposal of a segment (net of tax)
Net income

2,880
50
120
210
385

December 31, 2005

Cash
Trading securities
Accounts receivable (net)
Merchandise inventory
Tangible fixed assets
Total assets
Current liabilities
Total liabilities
Common stock outstanding
Retained earnings

2005
$32
169
210
440
480
1,397
370
790
226
381

2004
$28
172
204
420
440
1,320
368
750
210
360

The total debt-to-equity ratio for Ostrander Corporation in 2005 is


A. 1.30
B. 2.07
C. 3.49
D. 1.85
A. The total debt-to-equity ratio is calculated as the total liabilities of the company divided the equity of the
company, made of stock and retained earnings. The liabilities of the company were $790 and the equity was
$607 (made up of $206 of capital stock and $381 of retained earnings). Dividing $790 by $607 we get a total
debt-to-equity ratio of 1.30.
B. The total debt-to-equity ratio is calculated as the total liabilities of the company divided the equity of the company,
made of stock and retained earnings. This answer does not include the capital stock in the denominator.
C. The total debt-to-equity ratio is calculated as the total liabilities of the company divided the equity of the company,
made of stock and retained earnings. This answer does not include the retained earnings of the company in the
denominator.
D. The total debt-to-equity ratio is calculated as the total liabilities of the company divided the equity of the company,
made of stock and retained earnings. This answer reverses the formula and divides the equity by the debt.

Question 74 - CPA Released 1998 H3 - Earnings-based Analysis


In computing the weighted-average number of shares outstanding during the year, which of the following midyear
events must be treated as if it had occurred at the beginning of the year?
A. Purchase of treasury stock.
B. Sale of preferred convertible stock.
C. Declaration and distribution of stock dividend.
D. Sale of additional common stock.
A. The purchase of treasury stock is not treated as if it occurred at the beginning of the year.

(c) HOCK international, page 46

Part 1 : 11/11/10 07:46:17


B. The sale of preferred convertible stock is not treated as if it occurred at the beginning of the year.
C. Stock dividends (and stock splits) are treated as if they occurred at the beginning of the year.
D. The sale of additional common stock is not treated as if it occurred at the beginning of the year.

Question 75 - CMA 1293 2-14 - Other Analytical Issues


Selected data from Sheridan Corporation's year-end financial statements are presented below. The difference
between average and ending inventory is immaterial.
Current ratio
Quick ratio
Current liabilities
Inventory turnover (based on cost of goods sold)
Gross profit margin

2.0
1.5
$120,000
8 times
40%

Sheridan's net sales for the year were


A. $240,000
B. $800,000
C. $1,200,000
D. $480,000
A. This answer is the value for current assets. See the correct answer for a complete explanation.
B. We first need to start this question by calculating what the inventory balance is. If the current liabilities are
$120,000 and the current ratio is 2, that means that current assets are $240,000. Given a quick ratio of 1.5 and
current liabilities of $120,000, we know that the numerator of the quick ratio is $180,000. The difference
between the current ratio and the quick ratio is inventory, so the difference in the numerator of these two
items must be inventory. Thus, inventory is $60,000. If the inventory was turned over 8 times, that means that
the cost of goods sold was $480,000. We know that since 40% of the sales price was profit that 60% of the
sales price is cost of goods sold. Dividing $480,000 by the .6 cost ratio gives us sales of $800,000.
C. This answer is incorrect. See the correct answer for a complete explanation.
D. This answer is the value for cost of goods sold. See the correct answer for a complete explanation.

Question 76 - CMA 1280 4-4 - Short-term Liquidity


Depoole Company is a manufacturer of industrial products and employs a calendar year for financial reporting
purposes. Assume that total quick assets exceeded total current liabilities both before and after each transaction
described. Further assume that Depoole has positive profits during the year and a credit balance throughout the year
in its retained earnings account.
Obsolete inventory of $125,000 was written off during the year. This transaction
A. Decreased the current ratio.
B. Increased net working capital.
C. Decreased the quick ratio.
D. Increased the quick ratio.

(c) HOCK international, page 47

Part 1 : 11/11/10 07:46:17


A. The writing off of obsolete inventory will decrease the current ratio. This is because the inventory amount
is being reduced and there is no change to the current liabilities. This reduction in the numerator of the
current ratio will decrease the current ratio.
B. The writing off of obsolete inventory will decrease working capital because it reduces the amount of current assets
that the company has.
C. The writing off of obsolete inventory does not affect the quick ratio. This is because inventory is not included in the
calculation of the quick ratio.
D. The writing off of obsolete inventory does not affect the quick ratio. This is because inventory is not included in the
calculation of the quick ratio.

Question 77 - CMA 693 2-6 - Return on Invested Capital


Lisa, Inc.
Statement of Financial Position
December 31, 2002
(in thousands)
Assets
Current assets Cash
Trading securities
Accounts receivable (net)
Inventories (at lower of cost or market)
Prepaid items
Total current assets
Long-term investments Securities (at cost)
Property, plant, & equipment Land (at cost)
Building (net)
Equipment (net)
Intangible assets Patents (net)
Goodwill (net)
Total long-term assets
Total assets
Liabilities & equity Current liabilities
Notes payable
Accounts payable
Accrued interest
Total current liabilities
Long-term debt Notes payable 10% due 12/31/2005
Bonds payable 12% due 12/31/2004
Total long-term debt
Total liabilities
Equity Preferred - 5% cumulative, $100 par, nonparticipating, authorized, issued and outstanding, 1,000
shares
Common - $10 par 20,000 shares authorized, 15,000 issued and outstanding shares
Additional paid-in capital - common
Retained earnings
Total equity
Total liabilities & equity

2005
$30
20
45
60
15
170
25
75
80
95
35
20
330
$500

2004
$25
15
30
50
20
140
20
75
90
100
17
13
315
$455

$23
47
15
85
10
15
25
110

$12
28
15
55
10
15
25
80

100 100
150
75
65
390
$500

150
75
50
375
$455

Assuming that Lisa, Inc.'s net income for 2005 was $35,000, and there were no preferred stock dividends in arrears,
(c) HOCK international, page 48

Part 1 : 11/11/10 07:46:17


Lisa's return on common equity for 2005 was
A. 10.9%
B. 10.6%
C. 7.8%
D. 12.4%
A. This answer uses only the beginning common equity in the denominator, and not the average common equity.
B. The return on common equity is calculated as the income available for common shareholders divided by
the average common equity. The income for common shareholders is only $30,000. This is because $5,000 of
the $35,000 of income needs to be reserved for the cumulative preferred dividend. This is divided by the
average common equity which includes the common shares, the additional paid in capital and the retained
earnings. In total, this is $275,000 at the beginning of the year and $290,000 at the end of the year. In average,
this is $282,500. Dividing the income for common shareholders by the average common equity, we get 10.6%
C. This answer includes the preferred equity in the denominator of the calculation, instead of common equity.
D. This answer uses the entire net income in the numerator rather than the income available for common
shareholders.

Question 78 - CMA 0688 P4 Q19 - Earnings-based Analysis


Which one of the following items would likely increase earnings per share (EPS) of a corporation?
A. Purchase of treasury stock.
B. An increase in the common stock shares authorized to be issued.
C. Declaration of a stock dividend.
D. Declaration of a stock split.
A. The purchase of treasury stock decreases the number of shares that are outstanding. Since treasury
shares are not included in the EPS calculation, the purchase of treasury shares will increase EPS by
decreasing the number of shares to which the income must be distributed.
B. An increase in the number of authorized shares will have no impact on EPS because EPS is based on the number
of shares outstanding, not the number of shares authorized.
C. A stock dividend increases the number of shares outstanding and will therefore decrease EPS.
D. A stock split increases the number of shares outstanding and will therefore decrease EPS.

Question 79 - CIA 1194 IV-14 - Return on Invested Capital


If Company A has a higher rate of return on assets than Company B, this could be because Company A has a _____
profit margin on sales, or a _____ asset turnover ratio, or both.
A. Lower, Higher
B. Lower, Lower
C. Higher, Lower
D. Higher, Higher
A. Return on assets is calculated as the net income divided by sales. This can also be calculated by multiplying the
(c) HOCK international, page 49

Part 1 : 11/11/10 07:46:17


profit margin by the asset turnover ratio. Therefore, an increase in either one of these other ratios will cause an
increase in the return on assets.
B. Return on assets is calculated as the net income divided by sales. This can also be calculated by multiplying the
profit margin by the asset turnover ratio. Therefore, an increase in either one of these other ratios will cause an
increase in the return on assets.
C. Return on assets is calculated as the net income divided by sales. This can also be calculated by multiplying the
profit margin by the asset turnover ratio. Therefore, an increase in either one of these other ratios will cause an
increase in the return on assets.
D. Return on assets is calculated as the net income divided by sales. This can also be calculated by
multiplying the profit margin by the asset turnover ratio. Therefore, an increase in either one of these other
ratios will cause an increase in the return on assets.

Question 80 - CMA 1280 4-2 - Short-term Liquidity


Depoole Company is a manufacturer of industrial products and employs a calendar year for financial reporting
purposes. Assume that total quick assets exceeded total current liabilities both before and after each transaction
described. Further assume that Depoole has positive profits during the year and a credit balance throughout the year
in its retained earnings account.
The purchase of raw materials for $85,000 on open account would
A. Increase net working capital.
B. Decrease net working capital.
C. Decrease the current ratio.
D. Increase the current ratio.
A. Because this transaction increases current assets and current liabilities by the same amount, there is no change in
the level of working capital.
B. Because this transaction increases current assets and current liabilities by the same amount, there is no change in
the level of working capital.
C. Because Depoole has a current ratio of greater than one, the purchase of raw materials on account will
decrease the current ratio. This is because the purchase of materials on account will increase both the
current assets and the current liabilities by the same amount. Let us assume that Depoole had current assets
of $150 and current liabilities of $100, giving a current ratio of 1.50. Now let assume that they purchased $50
of materials. This increased the current assets to $200, and the current liabilities to $150. Now, the ratio is
1.33, which is lower than it had been.
D. Because the current ratio was greater than 1.0, this transaction will decrease the current ratio. See the correct
answer for a complete explanation.

Question 81 - CMA 693 2-2 - Short-term Liquidity


Lisa, Inc.
Statement of Financial Position
December 31, 2005
(in thousands)

(c) HOCK international, page 50

Part 1 : 11/11/10 07:46:17


2005 2004
Assets - Current
Cash
Trading securities
Accounts receivable (net)
Inventories (at lower of cost of market)
Prepaid items
Total Current Assets
Assets - Long-Term
Long-term investments Securities (at lost)
Property, plant & equipment Land (at cost)
Building (net)
Equipment (net)
Intangible assets Patents (net)
Goodwill (net)
Total Long-Term Assets
Total Assets
Liabilities - Current
Notes payable
Accounts payable
Accrued interest
Total current liabilities
Liabilities - Long-Term
Notes payable 10% due 12/31/2005
Bonds payable 12% due 12/31/2004
Total long-term debt
Total liabilities
Equity
Preferred - 5% cumulative, $100 par, nonparticipating, authorized, issued and
outstanding, 1,000 shares
Common - $10 par 20,000 shares authorized, 15,000 issued and outstanding shares
Additional paid-in capital - common
Retained earnings
Total Equity
Total Liabilities & Equity

$30
20
45
60
15
170

$25
15
30
50
20
140

25
75
80
95
35
20
330
$500

20
75
90
100
17
13
315
$455

$23
47
15
85

$12
28
15
55

10
15
25
110

10
15
25
80

100 100
150
75
65
390
$500

150
75
50
375
$455

Assume net credit sales and cost of goods sold for 2005 were $300,000 and $220,000 respectively. Lisa Inc.'s
accounts receivable turnover for 2005 was
A. 6.7 times.
B. 4.9 times.
C. 5.9 times.
D. 8.0 times.
A. This answer incorrectly uses ending receivables (instead of average receivables) in the numerator.
B. This answer includes only the year end receivables in the denominator (instead of the average receivables) and
uses cost of goods sold (instead of credit sales) in the numerator.
C. This answer incorrectly uses cost of goods sold (instead of credit sales) in the numerator.
D. Accounts receivable turnover is calculated as net credit sales divided by average accounts receivable.
Average accounts receivable was $37,500, and given information that credit sales were $300,000, we get a
receivables turnover of 8.

(c) HOCK international, page 51

Part 1 : 11/11/10 07:46:17

Question 82 - CMA 688 4-5 - Return on Invested Capital


The data presented below shows actual figures for selected accounts of McKeon Company for the fiscal year ended
May 31, 2004, and selected budget figures for the 2005 fiscal year. McKeon's controller is in the process of reviewing
the 2005 budget and calculating some key ratios based on the budget. McKeon Company monitors yield or return
ratios using the average financial position of the company. (Round all calculations to three decimal places if
necessary.)
5/31/05 5/31/04
Current assets
$210,000 $180,000
Noncurrent assets
275,000 255,000
Current liabilities
78,000 85,000
Long-term debt
75,000 30,000
Common stock ($30 par value) 300,000 300,000
Retained earnings
32,000 20,000
2005 Operations
Sales*
$350,000
Cost of goods sold
160,000
Interest expense
3,000
Income taxes (40% rate)
48,000
Dividends declared and paid in 2005 60,000
Administrative expense
67,000
*All sales are credit sales.
Composition of Current Assets
5/31/05 5/31/04
Cash
$20,000 $10,000
Accounts receivable 100,000 70,000
Inventory
70,000 80,000
20,000 20,000
Other
$210,000 $180,000
McKeon Company's total asset turnover for 2005 is
A. 0.722
B. 0.348
C. 0.805
D. 0.761
A. Total asset turnover is calculated as sales divided by average assets. This answer uses the year-end total assets
instead of the average total assets.
B. This answer is incorrect. See the correct answer for a complete explanation.
C. This answer is incorrect. See the correct answer for a complete explanation.
D. Total asset turnover is calculated as sales divided by average assets. Sales were $350,000 and average
assets were $460,000 ($485,000 in 2005 and $435,000 in 2004). This gives us $350,000 $460,000 = .761.

Question 83 - CMA 685 4-20 - Earnings-based Analysis


(c) HOCK international, page 52

Part 1 : 11/11/10 07:46:17


Baylor Company paid out one-half of last year's earnings in dividends. This year, Baylor's earnings increased by 20%,
and the amount of its dividends increased by 15%. Baylor's dividend payout ratio for the current year is
A. 47.9%
B. 50%
C. 57.5%
D. 78%
A. Probably the best way to solve this problem is to make an example. Let us assume that last year the
earnings were $100 and that $50 was paid out. If the earnings increased by 20%, the earnings are now $120. If
the payout increased by 15%, the payout is now $57.50. The new ratio is $57.50 / $120, or 47.9%. Also, given
that the payout increased by less than the earnings, we know that the payout ratio had to decrease, and this
is the only choice in which the answer decreased.
B. This is the payout ratio for the past year. See the correct answer for a complete explanation.
C. This answer is incorrect. See the correct answer for a complete explanation.
D. This answer is incorrect. See the correct answer for a complete explanation.

Question 84 - CMA 1291 P2 Q19 - Earnings-based Analysis


Sands, Inc. uses a calendar year for financial reporting. The company is authorized to issue 5,000,000 shares of $10
par common stock. At no time has Sands issued any potentially dilutive securities. Listed below is a summary of
Sands' common stock activities.
Number of common shares issued and outstanding at Dec. 31, 2003:1,000,000
Shares issued as a result of a 10% stock dividend on Sept. 30, 2004: 100,000
Shares issued for cash on March 31, 2005:
1,000,000
Number of common shares issued and outstanding at Dec. 31, 2005:2,100,000
A two-for-one stock split of Sands' common stock took place on March 31, 2006.
The weighted-average number of common shares used in computing earnings per common share for 2004 on the
2005 comparative income statement was
A. 1,025,000.
B. 2,100,000.
C. 1,100,000.
D. 1,050,000.
A. This answer assumes that the shares issued in the stock dividend were outstanding for only 3 months of the year,
instead of the entire year. However, stock splits and stock dividends that occur are considered to have been
outstanding from January 1 of the first year presented for comparison.
B. This answer incorrectly includes the shares that were issued for cash in 2005 as outstanding in 2004. See the
correct answer for a complete explanation.
C. In the 2005 financial statements, the 2004 EPS calculations will have to include the effect of any stock
splits or stock dividends that occurred in 2005. There were no stock splits or stock dividends in 2005 so the
weighted-average number of common shares outstanding for the 2004 EPS calculations on the 2005 financial
statements will be the same as it was on the 2004 financial statements. This was 1,100,000 which includes the
beginning 1,000,000 shares plus the 100,000 shares that were issued in September 2004 as a stock dividend.
Shares issued as a stock dividend are considered to have been outstanding from January 1 of the first year
(c) HOCK international, page 53

Part 1 : 11/11/10 07:46:17


presented (or January 1, 2004 for this specific question).
D. This answer assumes that the shares issued as part of the stock dividend were outstanding for only 6 months of
the year, instead of the entire year. However, stock splits and stock dividends that occur are considered to have been
outstanding from January 1 of the first year presented for comparison.

Question 85 - CIA 586 IV-24 - Return on Invested Capital


Return on investment may be calculated by multiplying total asset turnover by
A. Fixed-charge coverage.
B. Debt ratio.
C. Profit margin.
D. Average collection period.
A. This answer is incorrect. See the correct answer for a detailed explanation of how this problem should be solved.
B. This answer is incorrect. See the correct answer for a detailed explanation of how this problem should be solved.
C. Return on investment is calculated as the profit of the company divided by the average total assets. The
profit margin is calculated as profit divided by net sales. Total asset turnover is calculated as net sales
divided by average total assets. Putting all of the formulas together, if we multiply the total asset turnover by
profit margin we eliminate the net sales figure from each of the numbers and are left with the formula for
return on investment - profit divided by average total assets.
D. This answer is incorrect. See the correct answer for a detailed explanation of how this problem should be solved.

(c) HOCK international, page 54

S-ar putea să vă placă și